You are on page 1of 161

www.MRCPass.

com

Haematology


elzohryxp@yahoo.com https://www.facebook.com/elzohryxp

2102 May

www.MRCPass.com

Haematology

HaematologyQ001:

An8yearoldboywithsicklecelldiseasepresentswithbreathlessness.HisHbis4.5g/dl, WCCis3x10^9/landplateletcountis35x10^9/l. Whichorganismislikelytoberesponsible?


A.Coronavirus B.HIV C.HSV D.Parvovirus E.Epsteinbarrvirus

Answer:d)parvovirus. ParvovirusB19isthecommonestcauseofaplas ccrisisinsicklecellanaemia.Recovery shouldoccurwithin10dayswithconservativetreatment.

Dr.KhalidYusufElzohrySohagTeachingHospital2012 2

www.MRCPass.com

Haematology

HaematologyQ002:

A 45 year old man is being inves gated for easy bruising and malaise. Investigations reveal: Haemoglobin9.5g/dL Whitecellcount90x10^9/L Neutrophils45x10^9/L(1.57) Lymphocytes3.5x10^9/L(1.54) Myelocytes30x10^9/L Myeloblasts3x10^9/L Plateletcount750x10^9/L Whichofthefollowingdiagnosisislikely?
A.Acutemyeloidleukaemia B.Acutelymphocyticleukaemia C.Chronicmyeloidleukaemia D.Chroniclymphocyticleukaemia E.Polycythaemiarubravera

Answer:c)chronicmyeloidleukaemia. Ahighneutrophilcount,plateletcountpointstowardsmyeloidleukaemia. Acute leukaemia is dened as blast cells comprising 30% (in this case only 10% of myelocytes)ofthecelltype. HenceitmakesCMLmorelikelythanAML.

Dr.KhalidYusufElzohrySohagTeachingHospital2012 3

www.MRCPass.com

Haematology

HaematologyQ003: A30yearoldladya endsA&Ewithseverenosebleeds.Herinvestigationsshow: Hb10.5g/dl MCV80 WCC7x10^9/l platelets3x10^9/l Bloodfilmreport:Noplateletclumpsseen.Normalrbcandwbc Clottingscreennormal Whatisthemostlikelydiagnosis?
A.Thromboticthrombocytopenicpurpura B.Haemolyticuraemicsyndrome C.Acutelymphoblasticleukaemia D.Disseminatedintravascularcoagulation E.Immunethrombocytopenia

Answer:e)Immunethrombocytopenia. Asthereisnoabnormalityintheredandwhitebloodcellsonthebloodfilm,thisismost likelytobeimmunethrombocytopenia.Featuresconsistentwithadiagnosisofimmune thrombocytopenic purpura (ITP) are thrombocytopenia with plateletsbeing normal in size or may appear larger than normal, but uniformly giant platelets (approaching the sizeofredcells)shouldbeabsent.Themorphologyofredbloodcellsandwhiteblood cellsshouldbenormal.

Dr.KhalidYusufElzohrySohagTeachingHospital2012 4

www.MRCPass.com

Haematology

HaematologyQ004: A72yearoldladyhasrecentlybeenfoundtobeanaemic.Furtherbloodtestsshow: Hbof9.2g/dl WCCof8.0x10^9/l plateletcountof200x10^9/l MCVis104(8096) Ferri nis120g/l(15200) redcellfolateis350g/l(150650) B12is400pmol/l(120700) Bloodfilmshowsanisocytosisandpoikilocytosis Whichofthefollowingdiagnosisislikely?
A.Chroniclymphocyticleukaemia B.Autoimmunehaemolyticanaemia C.Sideroblasticanaemia D.Irondeficiency E.Lymphoma

Answer:c)sideroblasticanaemia. A high MCV with normal folate and B12 levels, normal iron and a blood lm showing anisocytosisandpoikilocytosissuggestssideroblasticanaemia.

Dr.KhalidYusufElzohrySohagTeachingHospital2012 5

www.MRCPass.com

Haematology

HaematologyQ005: A20yearoldmanpresentswithacuteseveredyspnoea.Hehadbeenstungbyawasp severalhoursago.Onexamination,hewashypotensiveandhadsignsofbronchospasm. Which one of the following investigations would confirm the type of hypersensitivity reaction?
A.Plasmatryptaselevel B.ESR C.SerumIgElevel D.Venomtoxinlevel E.ComplementC3level

Answer:C)serumIgElevel. TypeIhypersensi vityisoccurringinthiscaseofanaphylaxis.Ittakes30minutesfrom timeofexposureantigen.ThereactioninvolvesproductionofIgEwhichisreleasedfrom mastcells.

Dr.KhalidYusufElzohrySohagTeachingHospital2012 6

www.MRCPass.com

Haematology

HaematologyQ006: A 25 year old woman presents with diffuse lymphadenopathy, fever and malaise. Her bloodfilmshowsatypicallymphocytesandredcellagglutination. Whatisthemostlikelydiagnosis?
A.Legionella B.Infectiousmononucleosis C.Meningococcalmeningitis D.NonHodgkinslymphoma E.Autoimmunehaemolyticanaemia

Answer:b)Infectiousmononucleosis. InfectiousmononucleosisiscausedbyEpsteinBarrvirus.Itisoneofthecommoncauses ofatypicallymphocytes,alongwithcytomegalovirus,HIVandToxoplasma.Thefeatures oflymphadenopathyandatypicallymphocytessuggestinfectiousmononucleosis.

Dr.KhalidYusufElzohrySohagTeachingHospital2012 7

www.MRCPass.com

Haematology

HaematologyQ007:

A62yearoldmanwhowasasymptomatic,wasreferredforinvestigationofahighwhite cell count routinely found by the GP. On examination, he had palpable splenomegaly andlookedpale. Resultsreveal: Haemoglobin10.5g/dl(11.516.5) Plateletcount175x10^9/L(150400) Whitecellcount32x10^9/l(411) Neutrophilcount4x10^9/L(1.57) Lymphocytecount27x10^9/L(1.54) HisBloodfilmshowsmanymaturelymphocytes Whatisthebestinitialmanagement?
A.Prednisolone B.Periodofobservation C.Radiotherapy D.Splenectomy E.Chlorambucil

Answer:b)periodofobservation. In chronic lymphocytic leukaemia, Indications for therapy include fatigue, lymphadenopathy,anaemiaorthrombocytopenia. Alloftheothersaretreatmentoptionswhichcanbeused.

Dr.KhalidYusufElzohrySohagTeachingHospital2012 8

www.MRCPass.com

Haematology

HaematologyQ008: A 20 year old man presented with a cough and fevers. He was diagnosed as having a chest infection and was prescribed two different antibiotics. He felt worse two days laterandmentionedthathehaddarkurine. Investigationsshowed: Hb8.5g/dl MCV75 WCC12x10^9/l platelets155x10^9/l Bloodfilmshowed:anisopokilocytosisandbitecells Whatisthediagnosis?
A.G6PDdeciency B.Autoimmunehaemolyticanemia C.Aplasticanemia D.Immunethrombocytopenicpurpura E.Acutemyeloidleukaemia

Answer:A)G6PDdeciency Thereisevidenceofhaemolysis(bitecellsareschistocytes),inthiscasemostlikelydue toG6PDdeciency.DrugsnormallycausinghaemolysisinG6PDdeciencyaresulphur containingdapsone,antimalarials,bactrim/septrim,sulphonamides,primaquinine.

Dr.KhalidYusufElzohrySohagTeachingHospital2012 9

www.MRCPass.com

Haematology

HaematologyQ009: A65yearoldmanhasrecentlybeendiagnosedas havinga deep veinthrombosis.He also has symptoms of headaches and lethargy. On examination he was flushed. Investigationsreveal: haemoglobin19.5g/dL haematocrit0.6(0.40.52) whitecellcount10.5x10^9/L(411) plateletcount450x10^9/L(150400) Whichoneoffollowingisthemostappropriateinvestigation?
A.SerumEPOlevel B.Bonemarrowaspirate C.Neutrophilalkalinephosphatase D.Redcellmass E.SerumvitaminB12levels

Answer:d)Redcellmass. Themostappropriateinitialinvestigationwillberedcellmassstudieswhichwouldwill distinguishbetweentruerelativepolycythaemiafromsecondarypolycythaemia. In Polycythaemia Rubra Vera, the serum EPO is low (an elevated EPO level suggests secondarypolycythaemia). Haematocrit is high as is the Hb concentration. Thrombocytosis and leukocytosis can occur.TheNAPscoreandB12levelsarefrequentlyincreased.

Dr.KhalidYusufElzohrySohagTeachingHospital2012 10

www.MRCPass.com

Haematology

HaematologyQ010: A55yearoldwomanwhohadacerebrovascularaccidenttenmonthago,wasreferred for investigation of recurrent episodes of proximal deep venous thrombosis (DVT) of lowerlimbsinthelastsevenmonths. Investigationsshow: hemoglobin7.9g/dl hematocrit25% meancorpuscularvolume99 meancorpuscularhemoglobin32pg whitebloodcells4x10^9.l platelets93x10^9/l re culocytes5.4% lactatedehydrogenase944UI/l totalbilirubin50umol/l. A bone marrow biopsy showed a slight hyperplasia of erythrocytic bone marrow cell line. UrineDipstickblood+++ Whatisthelikelydiagnosis?
A.Haemolyticuraemicsyndrome B.AntithrombinIIIdeficiency C.Paroxysmalnocturnalhaemoglobinuria D.ProteinCdeficiency E.ProteinSdeficiency

Answer:c)Paroxysmalnocturnalhaemoglobinuria. Paroxysmal nocturnal haemoglobinuria (PNH) is an aplastic anaemia like syndrome whichredcellsarepredisposedtocomplementlysisandresultanthaemolyticanaemia. ThereisapancytopeniaaswellasatendencytowardsBuddChiarithrombosis. The diagnostic test is the HAM test. Serum (which contains complements) is acidified (activates the complement pathway) and mixed with red cells which undergo lysis. Haemosiderinisabyproductofhaembreakdowncontainingiron.Excessamountsleads torenaldamage,andisalsolostintheurine. InPNH,thereisalossofanchorprotein(GPIglycosylphosphatidylinositol)whichhold dierent an gens e.g. CD59, CD14. These are regulatory proteins for the complement pathway.

Dr.KhalidYusufElzohrySohagTeachingHospital2012 11

www.MRCPass.com

Haematology

HaematologyQ011: A17yearoldpa enthassicklecelldisease.Hepresentsunwellwithabdominalpain.He alsohasrightsidedfacialweakness.Investigations: Hb7.5g/dl platelets140x10^9/l Urea8mmol/l crea nine100mol/l sodium141mmol/l potassium3.8mmol/l bilirubin45mol/l AST35U/l ALP105U/l Alb42g/l LDH1250U/l Bloodfilmshowssicklecells Whichofthefollowingisthemostimportantmanagement?
A.Ivfluids B.Ivantibiotics C.Diamorphineinjections D.Bloodtransfusion E.Exchangetransfusion

Answer:e)exchangetransfusion. When there is neurological damage or visceral sequestration crisis in sickle cell crisis, exchange transfusion is indicated. Exchange transfusion involves drawing out the patient'sbloodwhileexchangingitfordonorredbloodcells.Itcanbedonemanuallyor automatically with erythrocytapheresis. It prevents stroke and also may be used in patients with severe acute chest syndrome and to reduce the risk of iron overload in patientswhorequirechronictransfusiontherapy.Studiessuggestthatit mayimprove oxygenationandreducehemoglobinSlevels.

Dr.KhalidYusufElzohrySohagTeachingHospital2012 12

www.MRCPass.com

Haematology

HaematologyQ012: A35yearoldmanpresentswithpallorandbreathlessness.Bloodtestsshowanaemia withaHbof7.5g/dl.AbloodfilmshowsHeinzbodies. Whichoneofthefollowingdiagnosesismostlikely?


A.Autoimmunehaemolyticanaemia B.Sideroblasticanaemia C.G6PDdeciency D.Postsplenectomy E.Sicklecelldisease

Answer:c)G6PDdeciency. Heinzbodiesareprecipitated,denaturedHbwithinredcells.TheyarepresentinG6PD deciency.(FavabeanscausehaemolysisinG6PD'BeansmeansHeinz'mnemonic).

Dr.KhalidYusufElzohrySohagTeachingHospital2012 13

www.MRCPass.com

Haematology

HaematologyQ013: Which one of these patient's results is most likely to have a diagnosis of chronic lymphaticleukaemia?
A.Awhitecellcountof35x10^9/landimmaturelymphocyteswithprominentnucleoli intheperipheralblood B.Awhitecellcountof15x10^9/landmaturelymphocyteswithcleavednucleiinthe peripheralbloodfilm C.Awhitecellcountof65x10^9/lwithneutophils,myelocytesandpromyelocyteson thebloodfilm D.Awhitecellcountof25x10^9/landsmearcellsontheperipheralbloodfilm E.Awhitecellcountof6x10^9/l,andmaturelymphocyteswithpolarvilliontheblood film

Answer: d) A white cell count of 25 x 10^9/l and smear cells on the peripheral blood film. Chroniclymphaticleukaemiaischaracterisedbyalymphocytosis.Thebloodfilmshows maturelymphocyteswithsmearorsmudgecells(theyaresquashedcells).

Dr.KhalidYusufElzohrySohagTeachingHospital2012 14

www.MRCPass.com

Haematology

HaematologyQ014: A65yearoldwomanhasadiagnosisofchroniclymphocyticleukaemia(CLL).Duringone followupappointmentshementionsthatshehasgotprogressivelymorelethargic. Herinvestigationsshow: Hb7.5g/dl,MCV118,platelets180x10^9/l,lymphocytes43x10^9/l,re culocyte count10%. Whichtestismostlikelytogivethecorrectdiagnosis?


A.Folatelevel B.Marrowtrephine C.Serumelectrophoresis D.Ferritin E.Coomb'stest

Answer:e)Coomb'stest. AraisedreticulocytecountcouldhaveledtothehighMCV.Theclinicalpictureisoneof haemolysiswhichisoccasionallyseeninCLL.TheCoomb'stestwillhelptoconfirmthis.

Dr.KhalidYusufElzohrySohagTeachingHospital2012 15

www.MRCPass.com

Haematology

HaematologyQ015: A 28 year old man with glucose6phosphate dehydrogenase deficiency presents with fatigueandjaundice.Thefeaturesdevelopedfollowingapneumoniaaweekago. Whichofthefollowingislikelytobefound?
A.Lowmeancellvolume B.Positivedirectantiglobulintest C.Spherocytespresentonbloodfilm D.Haemoglobinuria E.Reducedreticulocytecount

Answer:d)Haemoglobinuria. The clinical scenario describes haemolytic anaemia. Haemoglobinuria is seen in haemolytic anaemia. Patient may present with fatigue and tiredness. Low mean cell volume would mean lack of reticulocytosis. This is unlikely, there is usually increased reticulocyte count in all haemolytic anaemias including G6PD deciency. Spherocytes are seeninhereditaryspherocytosisandtheantiglobintestispositive inautoimmune haemoly canaemia(notjustG6PDdeciency).

Dr.KhalidYusufElzohrySohagTeachingHospital2012 16

www.MRCPass.com

Haematology

HaematologyQ016: A 60 year old AfroCarribean man is referred with abdominal discomfort. On examination,hehasmassivesplenomegaly.TheFBCshows: Hb8.2g/dl WBC15x10^9/l Platelets110x10^9/l Bloodsmear:erythroblasticpicture Whichofthefollowingdiagnosesisthemostlikely?
A.Myelofibrosis B.Polycythaemiarubravera C.NonHodgkinslymphoma(NHL) D.Aplasticanaemia E.Chronicmyeloidleukaemia

Answer:a)Myelofibrosis. Myelofibrosisexistislikelytobethe casedescribedabove. Medianageatdiagnosisis about 60 years, and median life expectancy from onset of symptoms is 10 years. In contrast,acuteMFinadulthoodisarapidlyfataldisorderinwhichsplenomegalyisnot usually observed; bone marrow examination typically reveals numerous bizarre megakaryocytesandblasts.

Dr.KhalidYusufElzohrySohagTeachingHospital2012 17

www.MRCPass.com

Haematology

HaematologyQ017: A32yearoldwomanpresentstothecasualtywithworseningdyspnoeaover3weeks. Shehasnohistoryofjaundiceofanaemia. On examina on, she had a blood pressure of 125/65 mmHg. Her conjunctivae were pale. Abdominal examination was unremarkable and there was no splenomegaly. Investigationsshow: Hb6.5g/dl WBC13.5x10^9/l Plts255x10^9/l MCV105 LDH680IU/dl DirectCoomb'stestpositive Film:Spherocytes++,reticulocytes++ Whatshouldbethetreatmentforhercondition?
A.Ironreplacement B.Bonemarrowexamination C.VitaminK D.Immunosupressants E.B12andfolate

Answer:d)immunosupressants. This woman is most likely to have autoimmune haemolytic anaemia (anaemia, high LDH, spherocytes on the blood film), and positive Direct Coomb's test. Steroids, intravenousimmunoglobulinmaybeusedasfirstlinetreatment,andbloodtransfusion may be necessary. Autoimmune haemolytic anaemia can be due to immune disorders (SLE),toxicchemicalsanddrugs,(methyldopa,penicillin),antiviralagents(eg,ribavirin), physicaldamage,andinfections(infectiousmononucleosis).

Dr.KhalidYusufElzohrySohagTeachingHospital2012 18

www.MRCPass.com

Haematology

HaematologyQ018: A35yearoldmanpresentswithfeversandlymphadenopathy.Abonemarrowbiopsy wasdoneandconfirmsHodgkin'slymphoma. Whichoneofthefollowingformhasthebestprognosis?


A.Nodularsclerosing B.Lymphocytepredominant C.Lymphocytedepleted D.Mixedcellularity E.Promyelocytic

Answer:b)lymphocytepredominant. Hodgkin's lymphoma is rare in children. Nodular sclerosing is the commonest and lymphocytedepletedistherarestform.Thelymphocytepredominantformhasthebest prognosis,whilstthelymphocytedepletedformhastheworst.

Dr.KhalidYusufElzohrySohagTeachingHospital2012 19

www.MRCPass.com

Haematology

HaematologyQ019: A30yearoldmanpresentswithmalaiseandisfoundtobeanaemicclinically.Hisblood testsreveal: Hbof10.5g/dl WCC8x10^9/l plateletcount180x10^9/dl re culcytecount160x10^9/l(50100) Bilirubinis80mol/l AST30U/l ALP110U/l LDHus380U/l(10250) Bloodfilmshowsspherocytosis Whichofthefollowingtestsismostappropriate?
A.Directantiglobulintest B.G6PDac vity C.Hbelectrophoresis D.Urinaryhaemosiderin E.Methaemoglobinlevels

Answer:a)Directantiglobulintest. The blood tests with high bilirubin, reticulocyte count and high LDH suggests haemolysis.Spherocytesonbloodfilmsuggestshereditaryspherocytosis(HS).InHSthe redcellsaresmaller,rounder,andmorefragilethannormal.Theconditioniscommoner amongNorthernEuropeans.Thedirectantiglobulintestwillhelptoconfirmthis.

Dr.KhalidYusufElzohrySohagTeachingHospital2012 20

www.MRCPass.com

Haematology

HaematologyQ020: A40yearoldmanhaspresentedwithseizuresandhasaconfirmedcerebralinfarcton headscan.Heiscommencedonphenytoin.3weekslaterhepresentswithlethargy. His bloods show Hb 8.0 g/dl, MCV 95 , WCC 3.2 x 10^9/l, platelets 65 x 10^9/l, Re culocyte count 1%. Ham's test was negative. Bone marrowaspirate and trephine biopsyshowedmarkedhypocellularityofthemarrowwithsomelymphoidaggregates. Whatisthelikelydiagnosis?
A.Folatedeficiency B.Myelofibrosis C.Aplasticanaemia D.Multiplemyeloma E.Bonymetastasis

Answer:c)aplasticanaemia. The diagnosis is likely to be phenytoin related aplastic anaemia. MCV is normal and thereisalowreticulocytecountaswellashypocellularbonemarrow.Sideeffectsof phenytoin are cerebellar syndrome, phenytoin encephalopathy, psychosis, locomotor dysfunction, hyperkinesia, megaloblastic anemia, decreased serum folate level, decreasedbonemineralcontent,liverdisease,IgAdeficiencyandgingivalhyperplasia.

Dr.KhalidYusufElzohrySohagTeachingHospital2012 21

www.MRCPass.com

Haematology

HaematologyQ021:

A 70 year old woman was admitted to hospital with severe breathlessness. On examina onherbloodpressurewas100/55mmHgandshehadaraisedJVPby4cm. Chestxrayshowedmildpulmonaryoedema. Investigationsrevealed: Haemoglobin6.6g/dL MCV108fL MCH32.0pg Whitecellcount3.0x10^9/L Platelets75x10^9/L SerumvitaminB12normal Folate2(320)g/l Whatshouldbedone?
A.Treatcongestivecardiacfailurethentransfuse B.Immediatebloodtransfusion C.Serumelectrophoresis D.Ironreplacement E.B12andfolatereplacement

Answer:e)B12andfolatereplacement. Blood transfusion may worsen cardiac failure in this case. In patients who are folate deficienterythropoiesisrapidlyresolveswhensupplementsaregiven,andtransfusionis rarelyneededintheelderly(unlesstheanaemiaisverysevere).
Dr.KhalidYusufElzohrySohagTeachingHospital2012 22

www.MRCPass.com

Haematology

HaematologyQ022:

A35yearoldmanhasrecentlybeendiagnosedwithHodgkin'slymphoma. Inreviewinghissymptoms,whichoneofthefollowingindicatesthepoorestprognosisin Hodgkin'slymphoma?


A.Mediastinal,inguinallymphadenopathyandfever B.Mediastinallymphadenopathyandnightsweats C.Abdominalandinguinallymphadenopathy,andnightsweats D.Cervicalandmediastinallymphadenopathy E.Mediastinalandinguinallymphadenopathy.

Answer:a)mediastinal,inguinallymphadenopathyandfever. StageIIIdiseaseoccurswhenlymphnodesarepresentacrossbothsidesofdiaphragm, hence worse prognosis than when lymph nodes are localised to the same side of the diaphragm.PresenceofBsymptomsnightsweatsandfeversalsoworsenprognosis.

Dr.KhalidYusufElzohrySohagTeachingHospital2012 23

www.MRCPass.com

Haematology

HaematologyQ023:

A 25 year old man was admi ed with a 2 month history of rash, fa gue, intermi ent hemoptysis, and purpura, culminating in a seizure. On examination, widespread petechiaeandpurpurawithscleralicteruswerenoted. Therewasnolymphadenopathyorsplenomegaly. Investigationsshow: plateletcount310^9/L hemoglobin5.5g/dL meancorpuscularvolume90fL whitecellcount19.610^9/L urea16mmol/L crea nine270mol/L lactatedehydrogenase2200U/L totalbilirubin79mol/L haptoglobin6g/L Blood film shows anisocytosis, moderate to marked polychromasia, and slight to moderatepoikilocytosis,predominantlyschistocytes. Whatisthebesttreatmentoption?
A.Haemodialysis B.Azathioprine C.Plasmaexchange D.Bonemarrowtransplant E.Intravenousimmunoglobulins

Answer:c)plasmaexchange. Thrombotic thrombocytopenic purpura (TTP) is characterised by microangiopathic haemolysisandthrombocytopenia.ThereisaspectrumofpresentationswithTTPHUS. Neurological features are present in 60% of patients of TTP and renal failure is often associatedinHUS(haemolyticuraemicsyndrome).

Dr.KhalidYusufElzohrySohagTeachingHospital2012 24

www.MRCPass.com

Haematology

HaematologyQ024:

A8yearoldboypresentstohisGPwithlethargyandpallor.Hisinvestigationsshow: Hb5.5g/dl WBC2.7x10^9/l Plts42x10^9/l Neutrophils0.9x10^9/l Whatisthenextbestinvestigation?


A.Peripheralbloodimmunophenotyping B.Bonemarrowcytogenetics C.Haematinics D.Bonemarrowaspirateandtrephine E.ANAandRheumatoidfactor

Answer:d)Bonemarrowaspirateandtrephine. Pancytopeniamaybeduetobonemarrowfailure(aplasticanaemia)ortobonemarrow infiltration (leukaemia, lymphoma or nonhaemopoietic malignancy). Aplastic anaemia may be idiopathic or secondary to drugs, paroxysmal nocturnal haemoglobinuria or Fanconis anaemia. In a child of this age, leukaemia (ALL, AML) or aplastic anaemia wouldbethemostlikelycausesofpancytopenia.

Dr.KhalidYusufElzohrySohagTeachingHospital2012 25

www.MRCPass.com

Haematology

HaematologyQ025:

A38yearoldwomanpresentstothehaematologistforreviewasshehaslethargy.Sheis onirontablets.Herbloodresultsshow: Hb9.5g/dl MCV105 WCC7x10^9/l platelets218x10^9/l Bloodfilmshowsanisopoikilocytosisandpoikilocytosis Whatshouldbedonenext?


A.Intramuscularirontherapy B.Bloodtransfusion C.Erythropoietin D.Investigationforfolateeficiency E.Noimmediateaction

Answer:E)noimmediateaction. Thebloodfilmandpoorresponsetoirontherapysuggestssideroblasticanaemia. Sideroblasticanaemiaismanagedbyremovingtheprecipitatingfactorse.g.alcoholor myelodysplasia.

Dr.KhalidYusufElzohrySohagTeachingHospital2012 26

www.MRCPass.com

Haematology

HaematologyQ026:

A 23 year old man presents with jaundice during a planned holiday to Africa. He has beentakingmalarialprophylaxis.Heisapyrexialandapartfromlethargy,feelswell.He reportspassingdarkurineforthepasttwodays. Whatisthelikelycause?
A.Betathalassemia B.HaemolysisduetoG6PDdeciency C.Sicklecellcrisis D.Falciparummalaria E.HepatitisCinfection

Answer:b)HaemolysisduetoG6PDdeciency. G6PDdeciencyiscommonintheMediterraneanandAfricanpopulations.Inheritance is Xlinked. Intravascular haemolysis is usually precipitatedby oxidative stress, such as infections and drugs. The most common drugs implicated are antimalarials, dapsone andsulphonamides.

Dr.KhalidYusufElzohrySohagTeachingHospital2012 27

www.MRCPass.com

Haematology

HaematologyQ027:

A 20 year old man complains of intermi ent dark urine and abdominal pains. He is foundtohaveahaemoglobinof9.7g/dl,buttherestofthefullbloodcountisnormal. Whatisthemostlikelydiagnosis?
A.Autoimmunehaemolyticanaemia B.G6PDdeciency C.Paroxysmalnocturnalhaemoglobinuria D.Paroxysmalcoldhemoglobinuria E.Hereditaryspherocytosis

Answer:c)Paroxysmalnocturnalhaemoglobinuria. Paroxysmalnocturnalhaemoglobinuriaiscausedbyadefectintheformationofared cell surface protein anchor, called GP1. As a result of the lack of this surface protein anchor, the red blood cells are more sensitive to complement lysis. Patients have intravascularhaemolysis,leadingtohaemoglobinuria,andincreasedriskofthrombosis, oftenoccurringinthemesentericvesselsandtheportalvein.Treatmentissupportiveor withbonemarrowtransplantation.

Dr.KhalidYusufElzohrySohagTeachingHospital2012 28

www.MRCPass.com

Haematology

HaematologyQ028:

A 35 year old man has had a 4 day history of dark urine. He has recently been on an antibioticforapresumedurinarytractinfection. Hisbloodtestsshow: Hb5.0g/dl MCV103 MCHC34g/dl(3235) WCC8x10^9/l platelets130x10^9/l re culocytes160x10^9/l(50100) PT13s(11.515.5) APTT38s(3040) urea6mol/l crea nine90mol/l sodium140mmol/l potassium4mmol/l bilirubin48(122)mol/l AST18(131)U/l ALP150(20120)U/l albumin32g/l LDH1550U/l(10250) Bloodfilmshowsblistercells. Whatisthediagnosis?
A.Autoimmunehaemolyticanaemia B.Hereditaryspherocytosis C.Paroxysmalnocturnalhaemoglobinuria D.G6PDdeciency E.Porphyria

Answer:d)G6PDdeciency. The patient's clinical and laboratory findings (eg, markedly decreased hemoglobin and hematocrit levels with a markedly increased serum LDH activity), are characteristic of acuteoxidantdamagetotheredbloodcellsandhemolysisduetoglucose6phosphate dehydrogenase(G6PD)deciency. The blood lm in G6PD deciency shows blister cells (membrane protrusion) (Heinz bodies may also be seen when there is no haemolysis). Treatment is with blood transfusion,orinseverecases,exchangetransfusion. Hemolyticcrisisoccursonlyafterexposuretocertainoffendingagents,includingdrugs, infections, exposure to fava beans, and diabetic acidosis. Drugs associated with hemolysis in G6PD deciency include an malarials (Primaquine, pamaquine), sulphonamides (Sulphamethoxazole), nitrofurantoin, analgesics (acetaminophen, aspirin,phenacetin),isoniazid(INH),methyleneblue,andnalidixicacid.

Dr.KhalidYusufElzohrySohagTeachingHospital2012 29

www.MRCPass.com

Haematology

HaematologyQ029:

A32yearoldCypriotpa entisbeinginves gatedforanaemia.HehasaHbof7.5g/dl andMCVis70.Hisbrotherandsistersarealsoanaemic. Whichoneofthefollowingismostlikely?


A.IncreasedIgMbandonserumelectrophoresis B.Redcellsshowmarkedhypochromia C.SevereirondeficiencyduetoGIbleeding D.SevereB12deciencyduetoperniciousanaemia E.Severefolatedeficiencyduetoceliacdisease

Answer:B)redcellsshowmarkedhypochromia This patient is likely to have thalassaemia (probably major). Hb electrophoresis may show increased HbA2 in thalassaemia minor. The severe imbalance of globin chain synthesis (alpha >> beta) results in ineffective erythropoiesis and severe microcytic hypochromicanemia,theremayalsobeprecipitateswithindamagedredcells.

Dr.KhalidYusufElzohrySohagTeachingHospital2012 30

www.MRCPass.com

Haematology

HaematologyQ030:

A35yearoldmanhasknowntype1VonWillebrandsdisease. Priortosurgery,whichisthebesttesttoassessbleedingtendency?
A.Prothrombintime B.FactorVIIIantigen C.FactorVIIIlevels D.Bleedingtime E.Thrombintime

Answer:b)factorVIIIantigen. Bleeding time is usually prolonged, and does not provide quantification of bleeding tendency.FactorVIIIantigenmeasuresthepresenceofvWFandgivesagoodestimate oftendencytobleed.

Dr.KhalidYusufElzohrySohagTeachingHospital2012 31

www.MRCPass.com

Haematology

HaematologyQ031:

An19yearoldmanpresentstotheA&Ewithapetechialrashandplateletcountof5x 10^9/l. He is otherwise well. A diagnosis of idiopathic thrombocytopenic purpura is made. Whichofthefollowingstatementsistrue?
A.Thepatientshouldbegivenaplatelettransfusion B.Thepatientshouldbeobserved C.ThepatientshouldbetreatedwithAntiD D.Thepatientshouldbetreatedwithintravenousimmunoglobulin E.Thepatientshouldbecommencedonsteroids

Answer:e)Thepatientshouldbecommencedonsteroids. In younger patients with ITP, the disease usually remits spontaneously within several weeksandnotreatmentisusuallyrequiredunlessthereissignificantbleeding. However, after adolescence, the disease tends to run a chronic relapsing course and thereforerequirestherapy. First line therapy is oral steroids. Patients who are refractory to, or are intolerant of steroidsmayrespondtointravenousimmunoglobulins(IVIg)orantiD.
Dr.KhalidYusufElzohrySohagTeachingHospital2012 32

www.MRCPass.com

Haematology

HaematologyQ032:

A35yearoldmanhasafaintmaculopapularrashonhischestandafewsho ylymph nodes. His bloods show Hb 13.5 g/dl, WCC 14.0 x10^9/l, plts 300 x 10^9/l. Blood lm showsreactivelymphocytes. Whichofthefollowingdiagnosisislikely?
A.Tuberculosis B.Nonhodgkin'slymphoma C.HepatitisB D.Infectiousmononucleosis E.Pneumonia

Answer:d)infectiousmononucleosis. There are several reactive lymph nodes as well as reactive lymphocytes suggestive of Ebstein Barr virus infection / infectious mononucleosis. Other causes of reactive lymphocytesareCMVinfection,toxoplasmosisandHIV.

Dr.KhalidYusufElzohrySohagTeachingHospital2012 33

www.MRCPass.com

Haematology

HaematologyQ033:

A 65 year old woman has symptoms of easy bruising. She was referred by the GP for investigation.Onexamination,shehadsplenomegaly. Resultsshow: Haemoglobin6.5g/dL(11.516.5) Whitecellcount17x10^9/l(411) Plateletcount32x10^9/l(150400) Bloodfilmshowslymphocytosis,myeloblastsandpromyelocytes. Whichoneoffollowinginvestigationsisofprognosticvalueinthissituation?
A.Bloodfilm B.Bonemarrowaspirate C.Cytogenetickaryotyping D.Immunophenotyping E.Serumelectrophoresis

Answer:c)cytogenetickaryotyping. Cytogeneticmonitoringoftheclinicalcourseofacutemyeloidleukaemia(suggestedby blasts) is o en associated with a specic chromosomal change, ie, t(8;21) in M2. Establishmentofthechangeatdiagnosisallowsrecognitionoftheleukemiccellsinthe marrowwhenrelapseorresidualdiseaseistobeevaluated.Italsoprovidesaprognostic determinant.
Dr.KhalidYusufElzohrySohagTeachingHospital2012 34

www.MRCPass.com

Haematology

HaematologyQ034:

A52yearoldfemalepresentswithacutechestpainandbreathlessness.Thechestpains were pleuritic and started to develop a week ago. Examina on reveals prominent P2 and clear breath sounds. She had bilateral ankle oedema. A urine dipstick showed protein+++. Whichisthemostlikelyexplanationforthesefindings?
A.FactorVLeiden B.ReducedantithrombinIIIactivity C.ReducedlevelsofVonWillebrandsfactor D.Reducedddimerconcentration E.ReducedfactorVIII

Answer:B)reducedantithrombinIIIactivity. AT III deficiency is associated with venous thrombosis. In this case, the history is consistentwithaclinicaldiagnosisofpulmonaryemboliandrenalveinthrombosis.

Dr.KhalidYusufElzohrySohagTeachingHospital2012 35

www.MRCPass.com

Haematology

HaematologyQ035:

A 35 year old man has had allogeneic bone marrow transplanta on which is HLA matched.2weekslaterhedevelopsadiffuserashalloverhisbody,feelssickandvomits several mes.Histemperatureis38C.Bloodtestsshow: Hb11.0g/dl WCC3x10^9/l Neutrophils1.5x10^9/l platelets18x10^9/l PT18s(11.515.5) urea7mol/l crea nine70mol/l sodium135mmol/l potassium4mmol/l bilirubin28mol/l AST48U/l ALP155U/l albumin32g/l LDH550U/l Whichofthefollowingismostlikely?
A.Bonemarrowfailure B.Parvovirusinfection C.Leukaemicspread D.Aplasticanaemia E.Graftversushostdisease

Answer:e)graftversushostdisease. Therash,systemicsymptoms,derangedliverenzymespointtowardsGVHD.Tcellsfrom the donor are attacking the recipient. Treatment is with immunosuppression: ciclosporin,methylprednisolone,methotrexateorantithymocyteglobulin(ATG).

Dr.KhalidYusufElzohrySohagTeachingHospital2012 36

www.MRCPass.com

Haematology

HaematologyQ036:

A 25 year old woman presented unwell with diarrhoea occurring 5 mes a day for 4 days.Shehadnotpassedurineforaday. Investigations: Haemoglobin8.2g/dL Whitecellcount14.2x10^9/L Neutrophils10.5x10^9/L Platelets32x10/L Fibrinogen5g/dL Serumsodium138mmol/L Serumpotassium6.3mmol/L Serumurea38mmol/L Serumcrea nine450umol/L Serumalbumin29g/L DipstickurineBlood+Protein+ Whatisthediagnosis?
A.Idiopathicthrombocytopenicpurpura B.Myelodysplasticsyndrome C.Disseminatedintravascularcoagulation D.Haemolyticuraemicsyndrome E.Aplasticanaemia

Answer:d)Haemolyticuraemicsyndrome. ThemostlikelydiagnosisisHaemolyticuraemicsyndromeduetodiarrhoeaassociated with E coli infection. A stool sample for culture and blood film are important investigationstobeperformed.
Dr.KhalidYusufElzohrySohagTeachingHospital2012 37

www.MRCPass.com

Haematology

HaematologyQ037:

A 60 year old man has a several month history of lower back pain and weakness. His bloodtestsshow: Hb11.0g/dl MCV95 WCC2.5x10^9/l platelets130x10^9/l PT13s(11.515.5) APTT28s(3040) urea26mol/l crea nine280mol/l sodium138mmol/l potassium4mmol/l bilirubin38mol/l AST26U/l ALP150U/l albumin33g/l totalprotein95g/l Whatisthemostlikelydiagnosis?
A.Multiplemyeloma B.Metastaticbladdercarcinoma C.Lymphoma D.Paraglanglioma E.Chronicmyeloidleukaemia

Answer:a)multiplemyeloma. Multiplemyelomaismostlikely.Thereisraisedprotein(6080normalrange)indicated probably paraproteinaemia. There is also low white cell count due to bone marrow infiltration,andrenalfailure.Thesymptomsofbonepainalsosuggestsinfiltration.

Dr.KhalidYusufElzohrySohagTeachingHospital2012 38

www.MRCPass.com

Haematology

HaematologyQ038:

A 20 year old girl is being inves gated for anaemia. Her father has previously had a splenectomy.Herbloodfilmshowsspherocytesandanaemia. Inviewofthelikelydiagnosis,whichisthemostusefulinvestigation?
A.Reticulocytecount B.Mottcell C.Haemosidnerinuria D.Haptoglobin E.IgGandC3complement

Answer:E)IgGandC3complement. Thispatientislikelytohavehereditaryspherocytosisinviewofthefamilyhistoryand thatthefatherhashadsplenectomyastreatment.However,raisedreticulocytecount anddecreasedhaptoglobinsandincreasedhaemosidurinuriawillbeallbepresentasa singletest,unhelpful.TheDirectAntiglobulinTestisusedtodetectIgGorC3boundto the surface of the red cell. In this scenario, it is helpful to exclude autoimmune haemolyticanaemia,sincespherocyteswouldalsobepresentonthebloodfilminAIHA.
Dr.KhalidYusufElzohrySohagTeachingHospital2012 39

www.MRCPass.com

Haematology

HaematologyQ039:

A 34 year old lady has a past history of an episode of deep vein thombosis and two miscarriages.ShepresentsnowwithfurtherepisoderofDVT.Shehadathrombophilia screenandwasfoundtohaveapositiveanticardiolipinantibody. Whatisthebesttreatment?
A.Clopidogrel B.Warfarin3months C.Longtermlowmolecularweightheparin D.AspirinandWarfarin E.Lifelongwarfarin

Answer:E)Lifelongwarfarin ThispatientwithrecurrentDVTshasthepresenceoflupusanticoagulant.Sherequires lifelongwarfarintreatment.


Dr.KhalidYusufElzohrySohagTeachingHospital2012 40

www.MRCPass.com

Haematology

HaematologyQ040:

A 40 year old male who has a rheumatoid arthritis is admitted with a urinary tract infection. Resultsshow: haemoglobin7.5g/dL(1116) whitecellcount1.5x10^9/L(411) platelets70x10^9/L(150400) Whichoneofthefollowingdrugsisthemostlikelycauseofpancytopenia?
A.Azathioprine B.Cyclophosphamide C.Prednisolone D.Cyclosporin E.Chloroquine

Answer:a)Azathioprine. Azathioprine is a thiopurine analogue drug which is metabolised in the liver to mercaptopurine. The main side effects are bone marrow suppression (may lead to a pancytopenia)andalsodruginducedhepatitis.

Dr.KhalidYusufElzohrySohagTeachingHospital2012 41

www.MRCPass.com

Haematology

HaematologyQ041:

A 50 year old man presents with mul ple bruises in the arms a er working in the garden. Inves ga onsshowed:Hb13.2g/dl,WCC5x10^9/l,plateletcount5x10^9/l. A bone marrow examination showed normal numbers of megakaryocytes and a diagnosisofidiopathicthrombocytopenicpurpurawasmade. Whatisthemostappropriatetreatment?
A.Tranexemicacid B.Oralprednisolone C.Bloodtransfusion D.Platelettransfusions E.Intravenousimmunoglobulin

Answer:b)oralprednisolone. The most appropriate treatment for this patient who is symptomatic from ITP is oral prednisolone. If the bleeding becomes severe, then IV immunoglobulin should be consideredinadditiontothesteroids.

Dr.KhalidYusufElzohrySohagTeachingHospital2012 42

www.MRCPass.com

Haematology

HaematologyQ042:

An12yearoldgirlhasrecurrentepistaxis.Herinves ga onsshowHb11g/dl,Plts300x 10^9/l,PT16sec(1316sec),APTT95sec(2838sec). Whichofthefollowingdeficienciesismostlikely?


A.FactorVdeficiency B.FactorVIIdeficiency C.VonWillebrand'sfactor D.Anticardiolipinantibody E.FactorXdeficiency

Answer:c)vonWillebrand'sfactor. von Willebrand's disease would be most likely due to the prolonged APTT, the rest of thefactors(sameonesasthosewhichwarfarinacton)prolongPT.

Dr.KhalidYusufElzohrySohagTeachingHospital2012 43

www.MRCPass.com

Haematology

HaematologyQ043:

A70yearoldmanisonlifelongoralan coagula onforrecurrentDVT.Hepresentswith minor bleeding from his gums for 1 day. His INR is 9.0. All other investigations are normalandheisotherwisewell. Whatisthemostappropriatecourseofaction?
A.Stopwarfarin,monitorINRandrestartwhenINR<5.0 B.StopwarfarinandadministerVitaminK2mg C.StopwarfarinandinstituteeitherLMWheparin D.StopwarfarinandgiveFFP E.Reducedoseofwarfarinto0.5mgun lINRnormalises

Answer:a)Stopwarfarin,monitorINRandrestartwhenINR<5.0. Ifthereisonlyminorbleeding,thencessationofwarfarinisallthatisrequired.Ifthere are other risk factors or if there is major bleeding then the use of vitamin K or fresh frozenplasmashouldbeconsidered.

Dr.KhalidYusufElzohrySohagTeachingHospital2012 44

www.MRCPass.com

Haematology

HaematologyQ044:

A 65 year old lady has a diagnosis of Non Hodgkin's lymphoma and has recently commenced chemotherapy. She now complains of feeling breathless and unwell. On examination,sheispaleandslightlyjaundiced.Shehassplenomegaly. Investigationsshow: Hb3.5g/dl MCV106 WCC8x10^9/l platelets250x10^9/l Re culocytes125x10^9/l(N2090) HerFBCprechemotherapywasnormal. Whatisthemostlikelyexplanationforthis?
A.Paroxysmalcoldhaemoglobinuria B.Bonemarrowsuppression C.Megaloblasticanaemia D.Autoimmunehaemolysis E.Paroxysmalnocturnalhaemoglobinuria

Answer:d)Autoimmunehaemolysis. Anaemia,raisedMCVandhighreticulocytecountsuggestshaemolysis.Thismayoccur secondarytoNHL.Inaddition,thereisastrongassociationwithfludarabine.

Dr.KhalidYusufElzohrySohagTeachingHospital2012 45

www.MRCPass.com

Haematology

HaematologyQ045:

Apatienthashadasplenectomybecauseofhereditaryspherocytosis. Howlongshouldpenicillinprophylaxisbeused?
A.Duringacuteinfections B.1year C.10years D.15years E.Lifelong

Answer:e)lifelong. Followingsplenectomy,patientsshouldreceivelifelongpenicillinprophylaxis.Themajor complicationofsplenectomyisoverwhelmingsepsiswithencapsulatedbacteria(eg,S pneumoniae,Hinfluenzae,Nmeningitidis).Theoverallriskofsepsisinasplenicpatients isapproximately2%butvariesdependingontheageandunderlyingdiseases.

Dr.KhalidYusufElzohrySohagTeachingHospital2012 46

www.MRCPass.com

Haematology

HaematologyQ046:

A13yearoldchildhashadrecurrentepisodesofbonepain.Hehasbeenadmittedto hospital several times due to severe pains in the last 5 years. He has X rays which shownecrosisofthehip. Whichofthefollowingdiagnosisislikely?
A.Multiplemyeloma B.Paget'sdisease C.Osteopetrosis D.Sicklecelldisease E.Thalassemia

Answer:d)sicklecelldisease. Aseptic necrosis of the hip, cholecystitis, renal papillary necrosis and proliferative retinopathyareclinicalfeaturesofsicklecelldisease.

Dr.KhalidYusufElzohrySohagTeachingHospital2012 47

www.MRCPass.com

Haematology

HaematologyQ047:

A20yearoldwomanwithsicklecellanemiapresentswithacuteshortnessofbreath.A chest x ray obtained with a portable unit initially showed no abnormalities except for bibasilarhazyopacities. Five and a half hours after admission, her oxygen satura on decreased to 76 percent with a respiratory rate of 24 breaths per minute. A repeat chest radiograph revealed increasedinterstitialmarkings. Howshouldshebetreated?
A.Antibioticsandfluids B.Intubationandventilation C.Noninvasiveventilationandplasmaexchange D.Splenectomy E.Highflowoxygen

Answer:c)noninvasiveventilationandplasmaexchange. This is a case of acute chest syndrome related to sickle cell anaemia. Non invasivve ventilation(CPAP)andplasmaexchangeisthebestoption,oftenalongwithantibiotics becausethechestsyndromecanbeprecipitatedbyinfection.

Dr.KhalidYusufElzohrySohagTeachingHospital2012 48

www.MRCPass.com

Haematology

HaematologyQ048:

A 25 year old female has had her rst DVT when she started taking the oral contraceptivepill.SherevealsthathermotherhasalsohadDVTbefore. Whichofthefollowingisshelikelytohave?
A.FactorVleidendeficiency B.ProteinCdeficiency C.ProteinSdeficiency D.AntithrombinIIIdeficiency E.Lupusanticoagulant

Answer:a)factorVleidendeficiency. Although they are all possibilities, the family history suggests factor V leiden or antithrombin III deficiency. A female who has DVT precipitated by the OCP suggests factorVleidenmoresothanantithrombinIII(malewouldsuggestthis).

Dr.KhalidYusufElzohrySohagTeachingHospital2012 49

www.MRCPass.com

Haematology

HaematologyQ049:

A32yearoldladyhadanewbornbabywithmarkedjaundice.Serumbilirubinwas359 mmol/Landhaemoglobinlow.Themotherhashadonepreviousnormaldelivery. Haemolyticdiseaseofthenewbornwassuspected. Whichofthesestatementsislikely?


A.FatherisORhve B.FatherisABRh+ve C.MotherisABRh+ve D.FatherisABRhve E.MotherisORh+ve

Answer:b)FatherisABRh+ve ToAnswerthisquestion,ABOgroupislessrelevantandRhstatusisrelevant. Rh grouping of foetus is decided by the Rh status of the father. The first child would have been Rh +ve and led to sensitisation (antibodies developed by the mother). The mother is Rh ve and the father is Rh +ve (who could also be homozygous or heterozygousforRh).

Dr.KhalidYusufElzohrySohagTeachingHospital2012 50

www.MRCPass.com

Haematology

HaematologyQ050:

A35yearoldmancomplainsoflegcrampsandisgivenquinineforthefirsttimebyhis GP. He then presents unwell and complains of dark urine. his Hb is 7.4 g/dl. Direct antiglobulintest(DAT)testisnegative. Whichofthefollowingislikely?
A.G6PDdeciency B.Autoimmunehaemolyticanaemia C.Paroxysmalcoldhaemoglobinuria D.Phosphokinasedeficiency E.Sicklecelldisease

Answer:a)G6PDdeciency. The direct antiglobulin test (Coomb's) is negative and suggests that this is not autoimmune since there is no antibody opsonisation on red cells. Quinine can precipitatehaemolysisinG6PDdeciencyascanaspirin,sulphonamides,favabeansand antimalarialagents.

Dr.KhalidYusufElzohrySohagTeachingHospital2012 51

www.MRCPass.com

Haematology

HaematologyQ051:
A40yearoldladywithpresentswithchronicdiscomfortinleftupperquadrantofthe abdomen.Investigationsshow: Hb16.9g/dl MCh55(2832)pg MCV69 Whitecellcount11x10^9/l Platelets490x10^9/l Whatistheunderlyingcause?
A.Essentialthrombocythaemia B.Primarypolycythaemia C.Renalcellcarcinoma D.Myelodysplasia E.Chronicmyeloidleukaemia

Answer:B)primarypolycythaemia. The raised Hb, white cell and platelet count are consistent with polycythaemia rubra vera. Splenomegaly is common, and occasionally splenic infarction as well which may lead to left upper quadrant pains. Essential thrombocythaemia is associated with anaemia.
Dr.KhalidYusufElzohrySohagTeachingHospital2012 52

www w.MRCPass.c com

Haem matology

Hae ematology Q052: y


A 30 year old man prese ents with r reddish urin and paleness. He mentions r ne recently trav vellingtoAs siaandbein ngonmalar rialprophyl laxis.Onph hysicalexam minationhe ehadan o axill larytemper ratureof36 C,pulse 6.5 eof120/min,respiratoryrateof 28/minandblood pres ssureof80/ /60mmHg.Otherphys sicalnding gswerenormal. Laboratory inv ga ons revealed h ves hemoglobin of 4.5 gm/ /dL, and platelet coun 342 x nt ^9/l. 10^ ipheralsme earshowed danisocytos sis(+),poikilocytosis(+),spherocyt tosis(),heinzbody Peri (+), ,sickling()andre culo ocytecount t6.2%. Otherresults: aspartatea aminotransf ferase420 iu/L,alanin neaminotra ansferase104iu/L, indirect bilirub 5.2 mg/ bin /dL, lactate dehydroge enase 721 i iu/L. Direct Coombs test was t neg gative. Urin neanalysis:redcolored d,protein(+ +),billirubin n(+++). Wha atisthemo ostlikelydia agnosis?
A.Haem molyticuraem micsyndrome B.Sickle ecelldisease e C.Malar ria D.G6PD Ddeciency E.Hered ditarysphero ocytosis

Answer:d)G6P PDdecienc cy. Pati ients with G G6PD defic ciency are o often asym mptomatic. Chronic haemolysis does not occu ur,buthae emolysisrat therisprecipitatedby drugssuch haschloroq quine,prim maquine, vitamin K, sulp phonamides G6PDH c s. catalyzes th synthesis of NADPH from the hexose he s H mon nophosphat pathwa Deficien te ay. ncy leads to oxidative daman nge and r red cell haemolysis.

Acutehemolys sisinG6PDd deciency,withtwo"b blistercells" "(arrows).

Dr.K KhalidYusuf fElzohrySo ohagTeachin ngHospital 2012 53

www.MRCPass.com

Haematology

HaematologyQ053:
An12yearoldboybleedsexcessivelyafteralaceration.Investigationsshow: Hb13.5g/dl WBC5.8x10^9/l Plts270x10^9/l PT15sec(1316sec) APTT85sec(2838sec) FactorVIIIandFactorIXlevels:normal APTT50:50mixwithnormalplasma:37sec Whichofthefollowingisthemostlikelydiagnosis?
A.HaemophiliaA B.FactorXIdeficiency C.FactorXdeficiency D.FactorXIIdeficiency E.FactorVIIdeficiency

Answer:b)FactorXIdeficiency. An isolatedprolonged APTT will becaused by deficiencies in factors VIII, IX, XI and XII and byvonWillebrandsdisease. FactorXIIdeficiencyisnotassociatedwithincreased bleeding. With normal Factor VIII and IX levels, this patient is most likely to have Factor XI deficiency.FactorXIdeficiencyisamildbleedingdisorderwithautosomalcodominant inheritance.DiagnosisisbyspecificFactorXIlevelestimation.
Dr.KhalidYusufElzohrySohagTeachingHospital2012 54

www.MRCPass.com

Haematology

HaematologyQ054:
A 48 year old man has a diagnosis of acute myeloid leukaemia. He was given chemotherapy. A few weeks into induction chemotherapy, he develops jaundice and fevers.Bloodculturesdidnotgrowanyorganisms.Despiteintravenousantibiotics,the patientremainedpyrexial. Whatisthelikelycause?
A.CMV B.Candidiasis C.Metastaticdisease D.Tuberculosis E.Rubella

Answer:A)CMV Thelikelycauseofpersistingpyrexiaplushepatitisinthisimmunocompromisedpatient aftertreatedwithappropriateantibioticswouldbeaCMVinfection.CMVinfectioncan also cause a pneumonitis and colitis. Treatment with an antiviral agent such as ganciclovircouldbeconsidered.
Dr.KhalidYusufElzohrySohagTeachingHospital2012 55

www w.MRCPass.c com

Haem matology

Hae ematology Q055: y


A 60 year old wo oman pre esents wi ith cervic cal lymphadenopathy and hep patosplenom megaly.Inve estigationsreveal: Hb1 10.5g/dl WBC4.6x10^9 9/l ts125x10^ ^9/l Plat seru umelectrop phoresis:Ig gMparaprot teindetecte ed,IgAandIgGlevelsa arenormal ydiagnosisi is: Themostlikely
A.Mono oclonalgamm mopathyofu uncertainsig gnificance(M MGUS) B.Chron niclymphocy yticleukaemia C.Hodgkinslympho oma D.Waldenstromsm macroglobulin naemia E.Multiplemyeloma a

Answer:d)Waldenstroms smacroglob bulinaemia. . The fact that there is IgM paraprotein sugges Walden e sts nstroms ma acroglobulinaemia. Wal ldenstrom's smacroglob bulinemiais salymphop plasmacytic clymphoma a(invasion ofbone mar rrow ) that leads to secretion of IgM. In t ncreased blood viscos sity may re esult in thro ombotic ph henomenon weakness, cryoglo n, obulinemia, neurologic disorders, and , fatig gue.

Mul ltipleplasm macytoidcellsinWaldenstrom'smacroglobuli inaemia

Dr.K KhalidYusuf fElzohrySo ohagTeachin ngHospital 2012 56

www w.MRCPass.c com

Haem matology

Hae ematology Q056: y


history of nigh sweats. On examin ht nation, he had lymph hadenopath palpable in the hy e cerv vicalregionandhepato osplenomeg galy. Inve estigations: hem moglobin8g g/dL leuk kocytecoun ntof6.6x10^9/l A lymph node biopsy o right neck was performed. It showed effaced normal of p d hitecturean ndclustersa andsheetsofReedSte ernbergcell ls arch Wha atisthemo ostlikelydia agnosis?
A.Essen ntialthrombo ocythaemia B.Acute emyeloidleu ukaemia C.Chron nicmyeloidleukaemia D.Hodg gkin'sdisease e E.Infect tiousmononucleosis

Answer:d)Hod dgkin'sdisease. Com mmonprese entingfeatu uresforHo odgkin'sdiseaeseareP PelEbstein fever,weig ghtloss, alco ohol induce pain and lymphad ed denopathy. Cold agglu utinins can occur, lea ading to possiblehaemo olyticanaem mia.

Ree edSternberg gcellinHod dgkin'sdise easewithlarge,promin nentnucleo oli.

Dr.K KhalidYusuf fElzohrySo ohagTeachin ngHospital 2012 57

www w.MRCPass.c com

Haem matology

Hae ematologyQ Q057: 5yearoldw womanhasahistoryof frecurrentanaemiawasnotedha avetargetc cellsand A45 How wellJollybo odiesonabloodfilmex xamination.Investigati ionsshow: Hae emoglobin7 7.8g/dL MCV V75 Seru umB12132 2ug/L(160 760) Seru umferri n9 9ug/L(153 300) Whi ichantibody yislikelyto obepresent t?
A.Antim mitochondria alantibody B.Intrinsicfactoran ntibody C.Antie endomysiala antibody D.Antit thyroidantib body E.Antig gastricpariet talcellantibo ody

MCH27 7pg(2832) Redcel llfolate90u ug/L(1606 640)

Answer:c)Antiendomysia alantibody. This patient h iron, fo s has olate and B deciency. This is most like due to coeliac B12 s ely dise ease. FBC shows ana C aemia in 50 of coeli disease pa ents; iron and folate decie 0% iac ency are both common (microcyte and mac es crocytes), h hypersegme ented leuco ocytes and Howell Jolly ybodies(hy yposplenism m). The preferred investiga e d ations are IgA Antitissue tran nsglutamina ase (tTG) or IgA End domysial (EMA) antibo odies. Antigliadin ant tibodies are less spec e cific, they can be positiveinCroh hn'saswell.

How wellJolly bodies are spherical b blueblack inclusions within red blood cel (iron lls defi iciency,thalassemia,postsplenec ctomy)

58

Dr.K KhalidYusuf fElzohrySo ohagTeachin ngHospital 2012

www.MRCPass.com

Haematology

HaematologyQ058:

A35yearoldwomanpresentswithjaundiceandlethargy.Investigationsreveal: Haemoglobin8.5g/dL(1116) Whitecells7x10^9/l Platelets190x10^9/l reticulocytecount130x10^9/L(2585) serumbilirubin55umol/L(120) Bloodfilmshowsspherocytes Whichofthefollowingshouldbedone?


A.Directantiglobulintest B.Ultrasoundoftheabdomen C.Bonemarrowbiopsy D.Bonemarrowaspirate E.G6PDenzymelevel

Answer:a)directantiglobulintest. One of the first tests to consider in a patient with haemolytic anaemia is the direct antiglobulintest(Coomb's).Thistoexcludeautoimmunehaemolyticanaemia.

Dr.KhalidYusufElzohrySohagTeachingHospital2012 59

www.MRCPass.com

Haematology

HaematologyQ059:

A42yearoldwomanhasahistoryofpositivelupusanticoagulant.Shehadapulmonary embolus diagnosed 8 years ago, and two presentations which were consistent with deepveinthrombosis6and12monthsgo. Whatisthebestmanagement?
A.Highdoseaspirin B.Lifelongwarfarin C.Antenataladvice D.Avoidanceoforalcontraceptivepill E.6monthsofwarfarinthenreassess

Answer:b)lifelongwarfarin. Morethanonethromboticeventwiththepresenceoflupusanticoagulantsuggeststhat thepatientrequireslifelongwarfarin.

Dr.KhalidYusufElzohrySohagTeachingHospital2012 60

www.MRCPass.com

Haematology

HaematologyQ060: A 65 year old woman has a haemoglobin of 5.5 g/dl. She has lethargy but no other symptoms.Herbloodfilmshowsovalmacrocytesandhypersegmentedneutrophils.She hasahistoryofhypothyroidismandisonthyroidreplacement.Shealsohasvitiligo. Whatisthemostlikelydiagnosis?
A.Multiplemyeloma B.Myelodysplasia C.Perniciousanaemia D.Irondeficiencyanaemia E.Haemolyticanaemia

Answer:c)Perniciousanaemia. Pernicious anaemia (PA) is a disease of the stomach that is characterised by megaloblas canaemiaduetovitaminB12deciency.Itissecondarytointrinsicfactor deficiencyandgastricatrophy.Itusuallyhasanautoimmunebasis.PerniciousAnaemia primarily affects the elderly most patients are over 60 years of age. Women are aectedmoreo enthanmen,inara oof3:2.Itmaybeassociatedwithautoimmune diseases,suchasAddison'sdisease,hypothyroidismandalsoanincreasedriskofgastric carcinoma.

Dr.KhalidYusufElzohrySohagTeachingHospital2012 61

www.MRCPass.com

Haematology

HaematologyQ061: A40yearoldladyhasbeenonwarfarinforpreviousDVT.Sheisnowbreathlessanda CTPAconfirmspulmonaryembolusdespiteherINRsbeingintherapeu crangeof23. Sheisalsohyponatraemicwithasodiumof129mmol/l. Withtheshortsynacthentest,shehasalowcor solof80nmolat0mingoingupto200 nmolat30min. Whichofthefollowingdiagnosisislikely?


A.Autoimmunepolyendocrinesyndrome B.Adrenaltumour C.ProteinCdeficiency D.FactorVleidendeficiency E.Presenceoflupusanticoagulant

Answer:e)presenceoflupusanticoagulant. Antiphospholipid syndrome is most likely due to the recurrent thrombotic tendency, lupus anticoagulant or anticardiolipin antibodies may be present. It has a propensity towardsadrenalveinthrombosisandcancausehypoadrenalismasinthiscase.

Dr.KhalidYusufElzohrySohagTeachingHospital2012 62

www.MRCPass.com

Haematology

HaematologyQ062:

A 20 year old girl receives a blood transfusion. 5 minutes a er the transfusion is commenced,shedevelopsatachycardiaandabdominalpains. Which of the following is the correct management of an acute haemolytic transfusion reactionduetoABObloodgroupincompatibility?
A.Stoptransfusionandassess B.Repeatcrossmatchandretransfuse C.Intravenousdextrose D.Hydrocor sone100mgintravenously E.Continuetransfusionslowly

Answer:a)Stoptransfusionandassess The immediate treatment of an acute haemolytic transfusion reaction due to a major blood group incompatibility is to discontinue the blood transfusion immediately. This should be followed by assessmentfor possible shock andresuscitation with fluids e.g. colloids.

Dr.KhalidYusufElzohrySohagTeachingHospital2012 63

www.MRCPass.com

Haematology

HaematologyQ063: A35yearoldmanhasrecentlybeencommencedonlowmolecularweightheparinand thenwarfarinfollowingadiagnosisofDVT. Whichofthefollowingiswellknownlongtermsideeffectofheparin?


A.Polycythaemia B.Visualloss C.Renalimpairment D.Osteoporosis E.Hirsutism

Answer:d)osteoporosis. Heparin induced thrombocytopenia, osteoporosis and thrombosis can occur. Warfarin cancauseskinnecrosis.

Dr.KhalidYusufElzohrySohagTeachingHospital2012 64

www.MRCPass.com

Haematology

HaematologyQ064: A patient who is known to have hereditary spherocytosis and has mild jaundice and gallstonesisawaitingsplenectomy. Howlongpriortosplenectomyshouldpneumococcalvaccinationbeadministered?
A.1day B.5days C.3weeks D.3months E.6months

Answer:c)3weeks. Pneumococcal immunisation should be administered to the pa ent 24 weeks before splenectomy.

Dr.KhalidYusufElzohrySohagTeachingHospital2012 65

www.MRCPass.com

Haematology

HaematologyQ065: A28yearoldwomanhashadadiagnosisofpulmonaryembolus.Shehasthefollowing investigations: Haemoglobin11.3g/dl,whitecellcount4.0x10^9/L,plateletcount45x10^9/L. Whichofthefollowingdiagnosesismorelikely?


A.Homocystinuria B.ProteinCdeficiency C.FactorVleidendeficiency D.Antiphospholipidsyndrome E.ProteinSdeficiency

Answer:d)antiphospholipidsyndrome. Apartfromathrombotictendency,antiphospholipidsyndromeisassociatedwithalow whitecellcountandthrombocytopenia.

Dr.KhalidYusufElzohrySohagTeachingHospital2012 66

www.MRCPass.com

Haematology

HaematologyQ066: A28yearoldladyis30weekspregnantwhenshepresentswithaleftsidedDVT.She hashadapreviousmiscarriagebefore.Herinvestigationsshow: Hb10.2g/dl MCV68 WBC8.0x10^9/l Plts250x10^9/l DirectCoomb'sTest:positive Re culocytecount90x10^9/l(25125) APTT51sec(normal2838sec) PT16sec(normal1316sec) ProteinCac vity0.75iu/ml(0.671.38) TotalproteinS100%(64154) Whichofthefollowingdiagnosisislikely?
A.Heparininducedthrombocytopenia B.ProteinCdeficiency C.ProteinSdeficiency D.FactorVleiden E.Antiphospholipidsyndrome

Answer:e)antiphospholipidsyndrome. Of the following choices, antiphospholipid syndrome is most likely because of the recurrentthrombotictendency.RaisedAPTTandpositiveDirectCoomb'stest(measures presenceofantibodiesonredcells)canbecausedbylupusanticoagulant.

Dr.KhalidYusufElzohrySohagTeachingHospital2012 67

www.MRCPass.com

Haematology

HaematologyQ067: A32yearoldwomanpresentswithbleedinggumsandeasybruising.Hermedications arelansoprazoleandcisprideforrefluxoesophagitis.Investigationsshow: Haemoglobin12.5g/dL(13.016.5) MCV90(8395) Platelets35x10^9/L(150400) Bloodfilm:occasionalgiantplatelets Prothrombin me12s(11.515.5) Whatisthelikelydiagnosis?
A.Disseminatedintravascularcoagulation B.Immunethrombocytopenia C.Thromboticthrombocytopenicpurpura D.Megakaryocyticthrombocytopenia E.Druginducedthrombocytopenia

Answer:b)Immunethrombocytopenia. This is a case of immune thrombocytopenia in which low platelets with other counts beingnormalapartfromslightanaemia(duetobleeding).

Dr.KhalidYusufElzohrySohagTeachingHospital2012 68

www.MRCPass.com

Haematology

HaematologyQ068: Achildhassevereanaemiaandbeendiagnosedashavingthalassemiamajor. Which of the following is the major form of haemoglobin present when the condition exists?
A.HaemoglobinA2 B.HaemoglobinC C.HaemoglobinF D.HaemoglobinH E.HaemoglobinA

Answer:c)HaemoglobinF. In Betathalassaemia major there is a complete defect in production of beta globin chains,whichleadstoimpairedforma onofHbA(whichismadeupof2alphaand 2 betachains).HaemoglobinFisthemajorhaemoglobinasthishaemoglobinismadeup ofalphaandgammachains. Patientswiththalassemiamajorrequirelifelongbloodtransfusions(hypertransfusions) withironchelationtherapy(desferrioxamine).

Dr.KhalidYusufElzohrySohagTeachingHospital2012 69

www w.MRCPass.c com

Haem matology

Hae ematologyQ Q069: 0yearoldm manwasre eferredfori investigatio onoflifelon nghemolytic canemia.Jaundice A20 acco ompaniedb byanemiaa andsplenom megalyhad dbeenappa arentsince earlylife.E Episodes of jaundice we more m ere marked duri infections or after fasting and less pron ing r nounced follo owingexposuretosunlight,wasc conspicuous s. Oninvestigatio onthefollow wingresults swereobta ained: moglobin,11 1.8g/dL hem mea ancorpuscu ularvolume e[MCV]85.5 5fL mea ancorpuscu ularhemogl lobin[MCH]29.1pg mea ancorpuscu ularhemogl lobinconce entra on[M MCHC]34g/ /dL re c culocyteco ount5.44 Bloo odfilmshow wsspherica alredcellsw withlackofpallorinthecentralar rea. Wha atisthedia agnosis?
A.Anaemiaofchron nicdisease B.Sidero oblasticanae emia C.Mega aloblasticana aemia D.Hered ditarysphero ocytosis E.Irond deficiencyan naemia

Answer:d)hereditaryspherocytosis. Her reditarysph herocytosisgeneforan nkyrin(cellmembraneprotein)ha asbeenmappedto chro omosome8 8andisaut tosomaldo ominant.It presentsinchildhood withjaund diceand sple enomegaly. Trea atmentisw withsplenec ctomy.

Red cells are m d more spher rical in her reditary sph herocytosis and lack the central area of pallor.

Dr.K KhalidYusuf fElzohrySo ohagTeachin ngHospital 2012 70

www.MRCPass.com

Haematology

HaematologyQ070:

A60yearoldmanhasbeenonwarfarinforaDVT.HehadanINRof9duringafollowup appointmentwhenhewasnoticedtohavehaematuriaandepistaxis. Whatistheappropriatemanagement?


A.2unitsbloodtransfusion B.2unitsofplatelets C.Freshfrozenplasma1unit D.Stopwarfarinandobserve E.0.5mgofvitaminKiv

Answer:e)0.5mgofvitaminKiv. The patient has minor bleeding. According to The British Society of Haematology guidelines,whenINRis>8withminorbleeding,warfarinshouldbediscontinueduntil theINRis<5.IVvitaminK0.5mgororalvitaminK5mgshouldthenbegiven.

Dr.KhalidYusufElzohrySohagTeachingHospital2012 71

www.MRCPass.com

Haematology

HaematologyQ071: A 55 year old man has had varicose vein repair. He develops a swollen leg five days followingsurgeryandanultrasoundofthelegconfirmsaDVT.Thisisdespitehimhaving prophylacticlowmolecularweightheparin. HisHbis11g/dl,WCC13x10^9/l,plateletcountis45x10^9/l. WhichofthefollowingcouldhavecausedtheDVT?
A.Behcet'sdisease B.FactorVleidendeficiency C.ProteinCdeficiency D.Immunethrombocytopenicpurpura E.Heparininducedthrombocytopenia

Answer:e)heparininducedthrombocytopenia. TypeIheparininducedthrombocytopenia(HIT)occurswithinafewdaysofheparinand isusuallymild. In this case, type II HIT is more likely, and this occurs slightly later (515 days). It is associatedwiththrombosisandalowplateletcount.Alternativeanticoagulationshould beused(hirudin,danaparoidsodium).

Dr.KhalidYusufElzohrySohagTeachingHospital2012 72

www.MRCPass.com

Haematology

HaematologyQ072: A55yearoldmanhasG6PDdeciency.Hepresentswithahaemoly ccrisisa eradrug wasstarted. Whichoneofthefollowingisprobable?


A.Carbamazepine B.Gentamicin C.Paracetamol D.Chloramphenicol E.Phenytoin

Answer:d)Chloramphenicol. Haemolysis in G6PD deciency is due to oxida ve damage (decreased genera on of NADPHduetoenzymedeficiency). Thecommoncategoriesofdrugsare: Sulphonamides Antimalarials Antipyretics(aspirin+paracetamol) Others:Chloramphenicol,nitrofurantoin,Dapsone,Probenecid,VitK

Dr.KhalidYusufElzohrySohagTeachingHospital2012 73

www.MRCPass.com

Haematology

HaematologyQ073: A32yearoldwomanwhois20weekspregnantpresentedwithlethargy,confusionand drow siness. On examination she has bilateral leg weakness and a purpuric rash was noticedonbothlegs.Investigationsshowed: Hb8.2g/dl WCC7.2x10^9/l platelets25x10^9/l re culocytes3% Urea28mmol/l Crea nine360umol/l Bloodfilmshowed:fragmentedcellsandpolychromasia Whattreatmentshouldbeadministered?
A.Dexamethasone B.Plasmaexchange C.Platelettransfusion D.Cyclophosphamide E.Bloodtransfusion

Answer:B)plasmaexchange. Thispatienthasthromboticthrombocytopenicpurpuraassuggestedbyhaemolysison thebloodfilm,anaemia,thrombocytopenia,renalfailureandalsoneurologicalfeatures. Treament of choice is plasma exchange with fresh frozen plasma infusion. High dose steroidsmayalsobebeneficial.Plasmaexchangeremovesantibodieswhichisthemain pathogenicprobleminthedisease.

Dr.KhalidYusufElzohrySohagTeachingHospital2012 74

www.MRCPass.com

Haematology

HaematologyQ074: A 23 year old woman presented very unwell with a miscarriage. On examination, she wasverypaleandbreathless.Shehadabloodpressureof90/60mmHg. Investigationsrevealed: Haemoglobin9g/dL Platelets52x10^9/L Prothrombin me20sec(1115) APTT55sec(3040) Fibrinogen0.3g/L(4) Bloodfilm:Fragmentedcells Whatshouldbeadministered?
A.Intravenoushydrocortisone B.VitaminC C.Tranexemicacid D.DDAVP E.Cryoprecipitate

Answer:e)cryoprecipitate. This is a case of disseminated intravascular coagulation (DIC) in which fibrinolytic systembecomesactivated,leadingtothrombinformation.Unregulatedfibrinolysisand systemicfibrinogenolysisoccurswithreleaseofplasminintothecirculation. Typically, the bloodfilm shows fragmented red blood cells. Treatment aims to correct thecoagulopathywithbloodproductse.g.cryoprecipitatetoreplacefibrinogen,orfresh frozenplasma.VitaminKcanalsobegivenineventofsignificantbleeding.

Dr.KhalidYusufElzohrySohagTeachingHospital2012 75

www.MRCPass.com

Haematology

HaematologyQ075: A55yearoldpa entisknowntohavealcoholicliverdisease.Hedrinks15pintsofbeer adayandhasoesophagealvariceswhenhehadendoscopy3monthsago.Henowhas melaenawiththefollowingbloodresults: Hb7.5g/dl MCV103 WCC11x10^9/l platelets100x10^9/l PT20s(11.515.5) APTT40s(3040) Fibrinogen0.8.0g/L(1.85.4) urea17mol/l crea nine105mol/l sodium130mmol/l potassium4mmol/l bilirubin62mol/l AST328U/l ALP200U/l albumin32g/l Apartfrombloodtransfusion,whichofthefollowingwouldbeuseful?
A.FactorVIII B.Cryoprecipitate C.Exchangetransfusion D.Haemodialysis E.Albumin

Answer:b)cryoprecipitate. Withalcoholicliverdisease,thereisaprolongedPTandlowplateletcount.However,in severe alcoholic liver disease fibrinogen can also be lowas in this case, thus cryoprecipitatewouldbeuseful.

Dr.KhalidYusufElzohrySohagTeachingHospital2012 76

www.MRCPass.com

Haematology

HaematologyQ076: A50yearoldmanpresentswithhypertension.Furtherbloodtestsrevealthefollowing: Hb18.6g/dl,WCC16x10^9/l,plts600x10^9/l.Theerythropoie nlevelisnormal. Whatisthemostlikelydiagnosis?


A.Secondarypolycythaemia B.Polycythaemiarubravera C.Myelofibrosis D.Gaucher'sdisease E.RecombinantEPOuse

Answer:b)polycythaemiarubravera. In polycythaemia rubra vera, the Hb, WCC and platelet counts are high along with a normalEPOlevel.EPOisraisedinsecondarypolycythaemia(e.g.hypoxia).

Dr.KhalidYusufElzohrySohagTeachingHospital2012 77

www.MRCPass.com

Haematology

HaematologyQ077: An 80 yearold lady complains ofmild breathlessness and lethargy.She men ons that she is a vegetarian. There is no history of haemetemesis or melaena. She has a past medicalhistoryofcongestivecardiacfailure. Onexamina on,sheispale,andhasvi ligoonherhands.ShehasaJVPof+4cmand fineinspiratorycrepitations. Herinvestigationsshow: Hb4.5g/dl MCV105 WBC3.3x10^9/l Plts120x10^9/l Whatisthemostimportantinitialmanagement?
A.Bloodtransfusion B.StartvitB12andfolicacid C.Redcellmassstudies D.Bonemarrowaspirate E.Schillingtest

Answer:b)StartvitB12andfolicacid. PancytopeniaandraisedMCVsuggestssevereB12orfolatedeciency.Vitiligoisalsoa clueastoautoimmunephenomenon,andperniciousanaemiamaybeassociated.Inthis lady,bloodtransfusionmayexacerbatecardiacfailure,andsheisnotactivelybleeding, hencereplacementofB12andfolateisabetteroption(symptomswillimprovewithin 12weeks). B12deciencycanoccurasaresultofperniciousanaemia(intrinsicfactordeciency), dietary e.g. vegetarian, Crohns disease, Serum folate level less than 5 ng/ml or serum vitaminB12levellessthan100pg/mlisdiagnostic. Folatedeciencyistreatedbygivingfolicacidorallyat1to5mgdaily.B12deciencyis usuallytreatedbyparenteraladministra onofB12.Thetreatmentscheduleconsistsof giving1000gcobalaminintramuscularlydailyfor1014daysfollwedby1000goncea week till hematocrit becomes normal followed by 1000 g once a month for life in patients with pernicious anemia or those with malabsorption. Therapeutic doses of folatewillcorrect thehematologicabnormalitiesduetocobalamin deficiencyalsobut the neurologic abnormallities can w orsen, it is best to give B12 rst or both B12 and folatebutneverfolatealone.
Dr.KhalidYusufElzohrySohagTeachingHospital2012 78

www.MRCPass.com

Haematology

HaematologyQ078: A50yearoldwomanpresentedwithafiveyearhistoryofpaininthemiddle ofboth feet.Shealsohadahistoryofbackpain,paininbothsidesofherhip,andpaininboth metacarpals.Shehadaserumferritinconcentrationof1087g/l,withnormalresultsin liver function tests and a normal glucose concentration, full blood count, and erythrocytesedimentationrate. Whatisthediagnosis?
A.Wilson'sdisease B.Ochronosis C.Marblebonedisease D.Haemochromatosis E.Thalassemia

Answer:d)Haemochromatosis. Haemochromatosishasanautosomalrecessivepatternofinheritanceandaects1in 250ofthenorthernEuropeanpopula on,withupto10%ofpeoplecarryingthegene. Inheritance of the disease has long been associated with the ssue type HLA A3. A specicmuta onofthegene,C282Yiscommon. Theclinicalpresentationofhaemochromatosisisvariableandnotconfinedtotheclassic triadofcirrhosis,diabetes,andskinpigmentation.Inthiscasethepresentationiswith arthropathy.

Dr.KhalidYusufElzohrySohagTeachingHospital2012 79

www.MRCPass.com

Haematology

HaematologyQ079: A56yearoldmanwasdiagnosedwithmyelofibrosis. Whichofthefollowingisthemostcommonpresentationofthedisease?


A.Bleeding B.Respiratorypain C.Hyperuricaemia D.Fatigue E.Bonepain

Answer:D)fatigue Clinicalfeaturesofmyelofibrosisinclude: usually develops in adults over age 50 patients commonly present with fatigue and weaknessspleenisoftenmassivelyenlargedhepatomegalyoccursinoverhalfofcases

Dr.KhalidYusufElzohrySohagTeachingHospital2012 80

www.MRCPass.com

Haematology

HaematologyQ080: A 60 year old woman was admi ed with a 10month history of rash, fatigue, intermittent hemoptysis, and purpura. On admission, widespread petechiae and purpura with scleral icterus were noted. No lymphadenopathy or splenomegaly was present. Investigationsshowed: plateletcountof310^9/L hemoglobinlevel5.5g/dL meancorpuscularvolume103fL whitecellcount3.610^9/L neutrophilsat0.67withoutmyeloidblasts reticulocytecountwas0.20 serumurea15.4mmol/L(43mg/dL) crea nine,239mol/L lactatedehydrogenase(LDH)2505U/L totalbilirubin19mol/L(4.6mg/dL) haptoglobin,6g/L Reviewoftheperipheralsmearrevealednotableredcellmorphologyand24nucleated redbloodcellsper100whitebloodcells,manyofwhichweredysplastic. Whatisthemostlikelydiagnosis?
A.Idiopathicthrombocytopenicpurpura B.Thalassemia C.Sicklecelldisease D.Myelodysplasia E.Chronicmyeloidleukaemia

Answer:d)myelodysplasia. ~Thereisagradualhistoryofprogressionandthepatienthasapancytopenia,sheisalso in the right age group for myelodysplasia. Myelodysplasia can be classified into five subtypes Refractoryanaemia Refractoryanaemiawithringsideroblasts Refractoryanaemiawithexcessblasts Refractoryanaemiawithexcessblastsintransformation(nearAML) CML. Few patients require aggressive therapy such as chemotherapy, it is reserved for younger patients to prevent progression to AML. Supportive therapy includes blood transfusions,platelettransfusionsorGCSFtoimprovebloodcounts.Howevermedian survivalisonly2years.

81

Dr.KhalidYusufElzohrySohagTeachingHospital2012

www.MRCPass.com

Haematology

HaematologyQ081: An18yearoldgirlpresentswithepistaxis.SheisfoundtohaveaprolongedAPTT.Her Motherhashadpreviousbleedingepisodeswithsimilarcoagulationtestresults. Whatisthemostlikelydiagnosis?


A.FactorVIIdeficiency B.FactorVdeficiency C.ProteinCdeficiency D.VonWillebrand'sdisease E.HaemophiliaB

Answer:d)vonWillebrand'sdisease. The APTT is a general clotting screen which detects defects in the intrinsic clotting pathway (factors XII, XI, IX, and VIII, to which Von Willebrand factor is linked). Von Willebrand's disease is a predominantly autosomal dominant condition which is associatedwithableedingtendency,whichisusuallymildandwithaprolongedAPTT. TheteststodiagnosevWDinclude: bleedingtime(prolonged) factorVIIIleveltest(measurestheleveloffactorVIIIanditsabilitytofunction) vonWillebrandfactorantigentest(thedisorderisconsideredmildifapersonhas20% to40%ofthenormalamount,severeiftheamountislessthan10%ofnormal) ristocetin cofactor activity test (measures how well the von Willebrand factor is working)

Dr.KhalidYusufElzohrySohagTeachingHospital2012 82

www.MRCPass.com

Haematology

HaematologyQ082: Apatientpresentswithacutepromyelocyticleukaemia. Whatisthelikelymechanismunderlyingleukaemogenesis?


A.Aberrantfusionof2genes B.Posttranslationalmodification C.Overexpressionofcellularoncogene D.Impaireddegradationofprotein E.Shorttelomere

Answer:A)aberrantfusionof2genes Acute promyelocy c leukaemia is frequently due to chromosomal transloca on t (15; 17).

Dr.KhalidYusufElzohrySohagTeachingHospital2012 83

www.MRCPass.com

Haematology

HaematologyQ083: A 15 year old male comes to the hematology clinic for his specialty care for von WillebrandDisease.ThepastmedicalhistoryrevealsthathewasdiagnosedwithType1 von Willebrand Disease as a toddler after abnormal bruising and prolonged bleeding wasnotedbyhisfamily. Whattreatmentisrecommendedifheweretohaveasignificantepisodeofbleeding?
A.Freshfrozenplasma B.FactorX C.FactorIX D.Aspirin E.DDAVP

Answer:e)DDAVP. Treatmentmay include Desmopressin (DDAVP),FactorVIII and tranexemic acid in von Willebrand'sdisease.

Dr.KhalidYusufElzohrySohagTeachingHospital2012 84

www w.MRCPass.c com

Haem matology

Hae ematologyQ Q084: A15 5yearoldb boypresent tedtoclinicwitha6m monthhistor ryofanorex xiaandmala aise.On examination,h hehadpalpa ableinguina allymphade enopathyandsplenom megaly. estigationss show: Inve Hb1 13.1g/dl Whitecellcoun nt20x10^9 9/l throcytesed dimenta on nrate15m mm/h eryt Peri ipheralbloo odfilmshow wed: atyp picallymphocytes,blas stcellsandneutropeni ia Wha atisthemo ostlikelydia agnosis?
A.Aplas sticanaemia B.Acute emyeloidleu ukaemia C.Chron nicmyeloidleukaemia D.Parox xysmalnoctu urnalhaemoglobinuria E.Acute elymphoblas sticleukaemia

Answer:e)acutelymphob blasticleuka aemia. Blas cells on t blood f st the film and a lymphocyto would suggest ac osis cute lympho oblastic leuk kaemia. The malignan cells are immature lymphoid blast cells. The patie with nt e . ent acut telymphob blasticleuka aemiaisusu uallyachild.

Bon neMarrowb biopsyshow wingpredo ominantlym mphoblasts

Dr.K KhalidYusuf fElzohrySo ohagTeachin ngHospital 2012 85

www.MRCPass.com

Haematology

HaematologyQ085: A25yearoldwomanhasthefollowinginvestigationsattheantenatalclinic: Hb10.3g/dl WBC5.6x10^9/l Plts290x10^9/l MCV69 MCH17.2 Iron20(1429)mol/l Ferri n150(15200)mol/l Whatisthemostusefulinvestigation?
A.Myelomascreen B.Haemoglobinelectrophoresis C.Folatelevels D.HbFlevel E.HbA2level

Answer:e)HbA2level. Iron deficiency anaemia and thalassaemia trait are the most likely diagnoses of microcytic anaemia. Iron deficiency is unlikely in this case in view of the iron studies beingnormal.BetathalassaemiatraitisdiagnosedbythepresenceofaraisedHbA2.If bothconditionsareexcluded,thenalphathalassaemiaisthemostlikelydiagnosis.

Dr.KhalidYusufElzohrySohagTeachingHospital2012 86

www.MRCPass.com

Haematology

HaematologyQ086: A 30 year old lady with von Willebrand's disease is due to have plas c surgery to her face and seeks advicefrom the haematologist. She mentions that she has a history of epistaxisandbleedinggums. Whichofthefollowingisthemostusefulassessmentofhercoagulationstatus?
A.Prothrombintime B.Activatedpartialthromboplastintime C.Thrombintime D.Bleedingtime E.FactorVIIIactivityassay

Answer:e)factorVIIIactivityassay. AlthoughbleedingtimeisprolongedinvonWillebrand'sdisease,thefactorVIIIactivity assay will give a measurement of the severity of her disease. The other useful tests would be the ristocetin cofactor assay and vWF antigen assays for von Willebrand's disease.

Dr.KhalidYusufElzohrySohagTeachingHospital2012 87

www w.MRCPass.c com

Haem matology

Hae ematologyQ Q087: A 20 year old pa ent has been found to have a medias nal mass o the ches X ray s e on st duringinvestig gationforapneumonia a. Heh hasaHbof12g/dl,WC CC180x10 0^9/l,platelets45x10^ ^9/l. Bloo odfilmshow wsblastcellswithhave eprominen ntnucleoli. The ereislittlecytoplasmw withinthew whitecells,a andthenucleoliareconvoluted. Whi ichisthelik kelydiagnos sis?
A.Acute emyeloidleu ukaemia B.Acute elymphoblas sticleukaemia C.Multiplemyeloma a D.Hodg gkin'slympho oma E.NonH Hodgkin'slym mphoma

Answer:b)acutelymphob blasticleuka aemia. The is a ver high white cell cou which should mak leukaem suspicio ere ry unt s ke mia ous. ALL occu ursinchildhoodandy youngadulthood.The bloodfilmd describeda abovedistin nguishes ALLfromAML(containselongatedinclusionscalledAuerro ods).

Mul ltiplelymph hoblastsinA ALL

Dr.K KhalidYusuf fElzohrySo ohagTeachin ngHospital 2012 88

www.MRCPass.com

Haematology

HaematologyQ088:

A 25 year old lady who was pregnant was treated for a deep vein thrombosis with intravenousheparin.Arecenttestshows: Haemoglobin10.2g/dL WhiteCellCount8x10^9/l Platelets32x10^9/l Whatisthebestcourseofactionthiswoman?
A.Changetoclexane B.Commencewarfarin C.Changetoaspirin D.Changetodanaparinoid E.Continueivheparin

Answer:D)changetoDanaparinoid ThispatienthasHeparinInducedThrombocytopaemia.WhenHITissuspected,heparin treatment should be converted to danaparoid , which is a low molecular weight heparinoid.Itisusuallygivenasanintravenousinfusion

Dr.KhalidYusufElzohrySohagTeachingHospital2012 89

www.MRCPass.com

Haematology

HaematologyQ089: A 65 year old man complains of breathlessness and redness. He has confirmed pulmonaryemboli.HisHbis18g/dl,WCCis15x10^9/landplateletcountis70010^9/l. Whichofthefollowingcanbehelpfulinconfirmingthediagnosis?
A.Bloodfilm B.Bonemarrowbiopsy C.Redcellmass D.NAPscore E.Kleihauertest

Answer:c)redcellmass. Thediagnosisispolycythaemiarubraveraandthiscanbeconfirmedbyaraisedredcell mass.NAPscoreisdecreasedinCML.AKleihauertestisusedtoconfirmtransplacental bloodlossfromfetustomother.


Dr.KhalidYusufElzohrySohagTeachingHospital2012 90

www.MRCPass.com

Haematology

HaematologyQ090:

A50yearoldmanwithnonHodgkin'slymphomaisonRituximab. WhichofthefollowingantigensdoesRituximabhaveanactionon?
A.CD8 B.CD8 C.CD19 D.CD20 E.CD154

Answer:d)CD20. Rituximabisanan bodytoCD20expressedonBcellsandisusedinBcelllymphomas (to try to cause cell lysis). The receptor is present in more than 90% of Bcell non Hodgkin's lymphomas. Molecules that a ach to CD20 can aect the growth and development of the tumor cells. Rituximab is an antibody that was developed using cloningandrecombinantDNAtechnologyfromhumanandmurinegenes.

Dr.KhalidYusufElzohrySohagTeachingHospital2012 91

www.MRCPass.com

Haematology

HaematologyQ091: A32yearoldwhoisknowntohaveITPpresentswithbleedingofhergums.Herplatelet countnormallyrunsat87x10^9/lbutnowhasdroppedto42x10^9/l. Whatisthebestmanagementplan?


A.Observation B.Steroids C.Platelettransfusion D.FFP E.Wholebloodtransfusion

Answer:B)steroids Chronic ITP rarely resolves spontaneously. First line treatment is with prednisolone. Patients with chronic ITP who require surgery may be given intravenous immunoglobulins which produce a transient rise in platelet count by blocking Fc receptorsonsplenicmacrophages. Platelet transfusion should be given only in lifethreatening haemorrhage to enhance haemostasis.

Dr.KhalidYusufElzohrySohagTeachingHospital2012 92

www.MRCPass.com

Haematology

HaematologyQ092: A45yearoldwomanpresentswithanupperGIbleedandrequiresabloodtransfusion. Halfwaythroughthefirstunitofblood,sheexperiencesgeneraliseddiscomfort. Whatisthemostappropriatecourseofaction?


A.ChestXrayandabdominalxray B.Bloodcultures C.Giveanalgesiaandcontinue D.IVsteroids E.StopthebloodtransfusionandgiveIVfluids

Answer:e)StopthebloodtransfusionandgiveIVfluids. Acutetransfusionreactionscancausegeneralizeddiscomfort,loinpainandpainatthe cannulasitemayallprecedehaemoglobinuriaandrenalfailure.Ifatransfusionreaction is suspected, the transfusion should be stopped immediately and IV fluids should be administered,topreventshock.

Dr.KhalidYusufElzohrySohagTeachingHospital2012 93

www.MRCPass.com

Haematology

HaematologyQ093: A55yearoldmanenquiresaboutrisksofbloodtransfusion. Whichofthefollowinginfectionsisbloodscreenedfor?


A.Varicellazoster B.HepatitisB C.Cytomegalovirus D.Malaria E.Salmonella

Answer:b)HepatitisB. CMV,malariaandsalmonellacanallbetransmittedbybloodproducts. IntheUK,routinetestingfordonorbloodisfor: HIV HepB&C Syphilis ABO+RhD

Dr.KhalidYusufElzohrySohagTeachingHospital2012 94

www.MRCPass.com

Haematology

HaematologyQ094: A 60 year old man presents with extensive bruising. He has a history of fa gue and dizzinessforthepastfewmonths.Onexamination,hehasapurpuricrashonhistrunk andlimbs.Investigationsshow: Hb7.5g/dl MCV105 WCC7x10^9/l platelets100x10^9/l Prothrombin me20(1217)s Fibrinogen90(150460)mg/dL Bloodlm:50%blastcells. Whatistheclinicalpictureconsistentwith?
A.Erythroleukaemicreaction B.Aplasticanaemia C.Disseminatedintravascularcoagulation D.Haemolyticanaemia E.Immunethrombocytopenicpurpura

Answer:C)disseminatedintravascularcoagulation. Theclinicaldiagnosisislikelytobeacutemyeloblasticleukaemia(AML).AMLsubtypes aredistinguishedfromotherrelatedblooddisordersbythepresenceofmorethan30% blasts in the blood, bone marrow , or both. One of the common complications is DIC, whichresultsinanelevatedprothrombintime,decreasedfibrinogenlevelandincreased fibrindegradationproducts.Acutepromyelocyticleukemia(APL),alsoknownasM3,is the mostcommonsubtypeofAMLassociatedwithDIC.Inleucoerythroblasticpicture, nucleatedredcellsandwhitecellprecursorsarefoundintheperipheralblood.

Dr.KhalidYusufElzohrySohagTeachingHospital2012 95

www.MRCPass.com

Haematology

HaematologyQ095: A60yearoldwomanhashadaprolongedITUstayduetoseverepneumoniaandsepsis requiring mechanical ventilation.She was noted to have worsening anaemiafollowing dischargefromITUat4weeks.HerHbis6g/dl,MCV109,WCC2.2x10^9/l,platelets 110x10^9/l. Whatisthelikelycauseofanaemia?
A.UpperGIbleed B.Aplasticanaemia C.AcutemyeloidLeukaemia D.Immunethrombocytopenicpurpura E.Acutefolatedeficiency

Answer:e)Acutefolatedeficiency. A patient who has been in intensive care for a significant period may not be getting enough folate, especially with increased needs for recovery. An acute deficiency state maythusdevelop.Thiswouldprecipitateapancytopeniaandmacrocyticanaemia.

Dr.KhalidYusufElzohrySohagTeachingHospital2012 96

www.MRCPass.com

Haematology

HaematologyQ096:

A 20 year old female presents with severe colicky abdominal pain, vomi ng and cons pa onof3daysdura on. Shehadaprevioushistoryofadmissiontohospitalwithsimilarfeatures. Herabdominalxrayandultrasoundscanwerenormal.Shewastreatedwithantibiotics, analgesics and antiemetics. Her urine was discoloured and she had a tonic clonic seizurewhilstontheward. Whatisthelikelydiagnosis?
A.Variegateporphyria B.Acuteintermittentporphyria C.Fabry'sdisease D.Gaucher'sdisease E.Matureonsetdiabetesoftheyoung

Answer:b)acuteintermittentporphyria. Acute intermittent porphyria is autosomal dominant disorder caused by a defect in porphobilinogendeaminaseactivity.Ifperipheralneuropathy,suchaspainintheback andlegsorparathesiasoccursitisalmostalwaysprecededbyabdominalpain. Other autonomic neuropathies that may be seen are sweating, vascular spasm, labile hypertension, and sinus tachycardia. Central nervous dysfunction can be seen as well withseizures,coma,bulbarparalysis,orcerebellarinvolvement. ThedefectinporphobilinogendeaminasecausesabuildupofALAandporphobilinogen (PBG)whichcausestheirincreasedsecretionintheurine.

Dr.KhalidYusufElzohrySohagTeachingHospital2012 97

www w.MRCPass.c com

Haem matology

Hae ematologyQ Q097: A 35 year old lady has ha frequent menorrha ad t agia over th last few months. Sh feels he he wellbutlookspaleonexa amination. Inve estigationsreveal: Hb8 8.6g/dl MCV V70 MCHC27(323 35)g/dl /l WCC7x10^9/ telets225x10^9/l plat Iron n9(1429) mol/l Ferr n12(15 ri 200)mol/ /l tota alironbindi ingcapacity y95(4572) )umol/l Wha atfeatureis slikelytobefoundonthebloodfi film?
A.Fragm mentedcell B.Helmetcell C.Polychromasia D.Spherocytes E.Targe etcells

Answer:E)targ getcells. This sladyhasir rondeficien ncyanemia.Commonb bloodfilmfe eaturesarepencilcells s,target cells sandpokilo ocytosis. Cau uses of tar rget cells are liver disease, post splene ectomy, iro deficien on ncy and thal lassemia.

Targ getCells

Dr.K KhalidYusuf fElzohrySo ohagTeachin ngHospital 2012 98

www w.MRCPass.c com

Haem matology

Hae ematologyQ Q098: A 5 year old man has a history of epistaxis. He is a 50 d also genera very tir ally red. On examination,h hehasnolymphoadeno opathyorsplenomegaly.Hisbloodtestsreve eal: Hb7 MCV90 7.6g/dl 0 MCHC32g/dl( (3235) WCC3x10^9/l Neu platelet utrophils1.5 5x10^9/l ts29x10^9 9/l urea a8mol/l crea nine125m mol/l sodium143mm mol/l mol/l potassium3.6mm rubin25m mol/l bilir AST18U/l ALP P150U/l albumin32g/l LDH H120U/l(10250) Whi ichofthefo ollowinginv vestigations swouldbem mosthelpfu ul?
A.CTofabdomen B.Geneticstudies C.PETsc can D.Bonemarrowasp pirationandc cytology E.Bonemarrowbiopsy

Answer:e)bon nemarrowb biopsy.

On the blood t tests, there is aplastic anaemia w e c which chou be due to infiltration by a uld tum mour,autoim mmuneord druginduce ed.Although hbothbonemarrowa aspirationis suseful, a bi iopsy is req quired to as ssess cellula arity and to exclude a o abnormal in nfiltration d to a due mlaignant pro ocess. In a aplastic ana aemia, ery ythropoietic cells, me c egakaryocyt tes and gran nulocyticce ellsareredu uced. Bon marrow aspiration involves a ne aspirating f fluid conten of mar nts rrow w hils bone st mar rrowbiopsy yinvolvesm moreforcetoobtaineb bonemorrowcore.See epictures:

Apla asticAnaem miaBoneM MarrowBiopsy


Dr.K KhalidYusuf fElzohrySo ohagTeachin ngHospital 2012 99

www.MRCPass.com

Haematology

HaematologyQ099: A45yearoldwomanpresentswithhaemetemesis.Shehasahaemoglobin(Hb)of4.5 g/dLandplateletcountof350x10^9/L. Whichofthefollowingisthemostappropriateproductordrugtouse?


A.Freshfrozenplasma(FFP) B.Ivmethylprednisolone C.Plasmaproteinfraction D.Packedredcells E.Ironinfusion

Answer:D)packedredcells. The patient who is anaemic and bleeding needs a blood transfusion with packed red cells(blood).Thisalsocontainssomewhitecells,platelets&asmallamountofplasma plus60100mlofaddi ve.

Dr.KhalidYusufElzohrySohagTeachingHospital2012 100

www.MRCPass.com

Haematology

HaematologyQ100: A75yearoldwomanwithchronic myeloid leukemia(CML),treated withhydroxyurea andinterferonfor12yearssufferedfromgradualdiseaseprogressionforoneyear. Investigationsshow: Haemoglobin11.6g/dL whitecellcount47x10^9/L(neutrophils,80%;lymphocytes,13%;metamyelocytes,6%; blasts,1%) plateletcount1220x10^9/L Whatshouldthepatientbetreatedwith?
A.Cyclophosphamide B.Prednisolone C.Radiotherapy D.Desferrioxamine E.Imatinib

Answer:e)Imatinib. Gleevec(imatinibmesylate,Novartis),isanoraldrugwhichinterfereswiththeactionof theabnormalBcrAbltyrosinekinaseinCMLwhitebloodcells. Before Gleevec, the most common drugs used to treat CML were the oral treatments hydroxyureaandbusulphan. Anintravenoustreatment,cytarabine,issometimesusedincombinationwithimmune therapy (interferon). Bone marrow or stem cell transplantation tends to be limited to youngerpatients.

101

Dr.KhalidYusufElzohrySohagTeachingHospital2012

www w.MRCPass.c com

Haem matology

Hae ematologyQ Q101: A30 0yearoldw womanisb beinginvest tigatedfor rightupperquadrant pains.She reports occa asionalepis sodesofjau undice,espe eciallyassoc ciatedwith infections.Hermothe er,sister andaunthasp previouslyhadgallstones.Onexam mination,sh hehassplen nomegaly.HerFBC isno ormal,buth herbilirubin nismildlye elevated. atisthelike elycausefo orherproba ablegallston nes? Wha
A.Sickle ecelldisease e B.Hered ditarysphero ocytosis C.Betat thalassemia D.Gilbert'ssyndrom me E.Parox xysmalnoctu urnalhaemog globinuria

Answer:b)Her reditarysph herocytosis. Her reditary sp pherocytosis is inher rited in an autosom domina n mal ant fashion It is n. characterised by increas sed red cell fragility. There is increased haemolysis during infe ections. Gal llstones are commonly associate Management is u e ed. usually supportive, alth houghsome ecasesrequ uiresplenec ctomytored ducetransfusionrequirements.

Sphericalredcellsinhereditaryspherocytosis

Dr.K KhalidYusuf fElzohrySo ohagTeachin ngHospital 2012 102

www.MRCPass.com

Haematology

HaematologyQ102: A 30 year old pa ent has signicant GI bleeding, but is concerned about the risks of bloodtransfusion. Whichofthefollowingisscreenedforindonatedblood?
A.JCvirus B.HumanTcellleukaemiavirus C.HIV1 D.NewvariantCJD E.Toxoplasmosis

Answer:c)HIV1. IntheUKeveryblooddonationistestedforevidenceofhepatitisB,hepatitisC,HIV1, HIV2andsyphilis.However,althoughtherearerecentconcernsregardingtranmission ofnewvariantCJD,therearenoreliablescreeningmethodsyet.

Dr.KhalidYusufElzohrySohagTeachingHospital2012 103

www.MRCPass.com

Haematology

HaematologyQ103:

A35yearoldwomanwithahaematologicalconditionhasbeentransfusedwithgroup specificplateletsonseveraloccasions.Herplateletcountdropsquickly5daysfollowing platelettransfusion. Whatshouldshebetreatedwith?


A.Freshfrozenplasma B.Cryoprecipitate C.Packedredcells D.Intravenousimmunoglobulin E.FactorVIII

Answer:D)intravenousimmunoglobulin. Post transfusion purpura is a transfusion reac on occurring occurs 5 14 days a er transfusionofplateletsorfreshfrozenplasma.Thisoccurswhenindividualslackingthe PLA1 an gen are transfused with blood containing PLA1 posi ve platelets. It is uncommonasonly23%ofthepopula onarePLA1nega ve. Treatment of choice is intravenous immunoglobulin or plasma exchange. Further platelettransfusionsshouldbewashedorbeHPA1Anegative.

Dr.KhalidYusufElzohrySohagTeachingHospital2012 104

www w.MRCPass.c com

Haem matology

Hae ematologyQ Q104: A70 0yearoldf femalepresentswith leftupper quadrantp painandmu ultipleecch hymoses (bru uising). Inve estigations: hem moglobin9.5 5g/dL plat teletcount30x10^9/L L whitecellcoun ntof8.2x10^9/L e y ellular (75% and show diffuse in %) ws nfiltration b large by The bone marrow biopsy is hyperce atyp picalcellsw withabundantcytoplasm. Wha atisthedia agnosis?
A.Acute emyeloidleu ukaemia B.Chron niclymphocy yticleukaemia C.Hodgkin'slympho oma D.Waldenstrom'sm macroglobulin naemia E.Polycy ythaemiarubravera

Answer:b)chro oniclympho ocyticleuka aemia. Chro onic lymph hocytic leuk kaemia aris from th neoplas prolifer ses he stic ration of re elatively mat ture lymph hocytes wh hich infiltra the blo ate ood, bone marrow o lymphor or reticular stru uctures. Mo are clon malignan ost nal ncies of B lymphocyte rather th T lymph es han hocytes. Bon nemarrowa aspiration/ /biopsytypicallythereisinfiltrat tionbylymp phocytes.

Diff fuseinfiltrat tionbylarge eatypicalcellsinCLL

105

Dr.K KhalidYusuf fElzohrySo ohagTeachin ngHospital 2012

www.MRCPass.com

Haematology

HaematologyQ105: A35yearoldmanwhoworksinafactoryhasaccidentallydrunkalargeamountofdye. Heisbroughttothehospitallookingabluecolour. HisHbis16g/dl.BloodgasesshowapHof7.37,pO2of13kPa,pCO2of4.5kPaandO2 satsof80%onthesatsmonitor. Whichisthediagnosis?


A.Methaemoglobinaemia B.Down'ssyndrome C.Fallot'stetralogy D.Carbonmonoxidepoisoning E.Polycythaemiarubravera

Answer:a)methaemoglobinaemia. Methaemoglobinaemiaisthemostlikelydiagnosisduetoingestionofanilinedyeinthis case. The pO2 is o en normal but the oxygen satura ons are reduced in methaemoglobinaemia.

Dr.KhalidYusufElzohrySohagTeachingHospital2012 106

www.MRCPass.com

Haematology

HaematologyQ106: A 40 year old woman has a long history of anaemia, and is not compliant with medications. Investigationsshow: Haemoglobin7.8g/dL(11.316.5) MCV85(8096) MCH26pg(2832) WCC7x10^9/l Platelets160x10^9/l SerumB12130ug/L(160760) Redcellfolate95ug/L(160640) Serumferri n11ug/L(15300) Whichofthefollowingantibodiesislikelytobepresentwiththecondition?
A.Antigliadinantibody B.Antiintrinsicfactorantibody C.Antiparietalcellantibody D.Antinuclearantibody E.Antiphospholipidantibody

Answer:a)antigliadinantibody. Therearemixedironandfolatedeficiencyduetocoeliacdisease.Thiscausesanormal MCV (dimorphic picture because of both micro and macrocytic features). Antibodies which are present in coeliac disease are : an endomysial and an gliadin. Vitamin B12 concentrations normalize on a glutenfree diet alone, but symptomatic patients may requiresupplementation.

Dr.KhalidYusufElzohrySohagTeachingHospital2012 107

www.MRCPass.com

Haematology

HaematologyQ107: A 60 year old lady is inves gated for recurrentepisodes ofgout.On examination, she lookedplethoricandhasa6cmsplenomegaly.Shehasthefollowingresults: Hb18.9gm/dl Hct0.612 Plateletcount468x109/l ESR1mm/1sthour coagulationscreennormal Whatisthediagnosis?
A.Essentialthrombocythaemia B.Chronicmyeloidleukaemia C.Polycythaemiarubravera D.Idiopathicthrombocytopenicpurpura E.Thalassemia

Answer:c)polycythaemiarubravera. ThecriteriaforPRVare: 1)increasedredcellmass 2)splenomegaly 3) increased platelets, leucocytes, INCREASED NAP score and B12 (increased B12 bindingproteinrelease). Gout occurs due to increased cell turnover, cerebral and myocardial ischaemia occurs duetofallinperfusionandraisedbloodviscosity. TheNAPscoreisasemiquantitativecytochemicalassessmentofalkalinephosphatasein neutrophils.TheNAPscoreisbasedonstainingintensity,withapossiblescoreof0400. It differentiates chronic myeloid leukaemia (low) from reactive leucocytosis (high), eg bacterialinfection.Itmayassistinthedifferentiationofpolycythaemiarubravera(high) fromothercausesoferythrocytosis(normal).

Dr.KhalidYusufElzohrySohagTeachingHospital2012 108

www w.MRCPass.c com

Haem matology

Hae ematologyQ Q108: A25 5yearoldw womanpre esentedtot thehemato ologyclinicw witha5yearhistoryo ofpallor and anemia. On physical examin nation, the patient was foun to hav mild e nd ve sple enomegaly. Resultsshow: Hem moglobin7.9g/dL plat teletcountof22610 0^9/L MCV V68 MCHwas20pg g(24.031.0pg) MCHC30.1g/d dL(32.036.0g/dL) umiron31.6mol/L(9 9.026.9) seru ri /mL(22400ng/mL) tran nsferrin161 1mg/dL(18 85370mg/ /dL) ferr n380ng/ Bon nemarrowa aspiratedemonstrated dsignificant thypercellu ularityassoc ciatedwithmarked eryt throidhype erplasia. Stainableiron storeswereincreased d.Astriking gfeaturewasthepres senceofnu umerous blas in which the perin sts nuclear iron granules encircled more than one third of the n nuclearcircumference. Wha atisthedia agnosis?
A.Haem mochromatos sis B.Sidero oblasticanae emia C.Multiplemyeloma a D.Waldenstrom'sm macroglobulin naemia E.Thalassemia

Answer:b)Side eroblasticanaemia. Side eroblastsar reabnorma alredcellprecursorsw withironloa adedmitochondria,fo orminga ring around th nucleus. Sideroblast anaemia is associa g he tic a ated with alcohol, lead drugs d, andmyelodysp plasia.Ther reisadefec ctinhaem synthesis,t thusexcess sloadingof fironto com mpensate i red ce precurso in ell ors and i into iron stores, so ometimes causing haemosiderosisinthelive er,anddesferrioxamine etherapym mayhelp. Ring siderob ged blasts are p precursors and hence are found in the bo marrow . Anti e d one w tube erculous drugs interfe with h ere haem metabolism by intereferin with pyr ng ridoxine availability.Som mecasesre espondtopyridoxineth herapy(not tpanthoten nicacid).

109

Dr.K KhalidYusuf fElzohrySo ohagTeachin ngHospital 2012

www.MRCPass.com

Haematology

HaematologyQ109: A15yearoldFilipinogirlisnotedtohaveahemoglobinof10.6g/dlwithanMCVof65 onroutinetesting.Shereportsregularmenseslas ng45dayseachcycle.Shehasno specificcomplaints.Sheisunawareofafamilyhistoryofanemia.Onexamination,there isnohepatosplenomegaly,jaundice,orscleraljaundice. Whatisthelikelydiagnosis?


A.Irondeficiency B.Leadpoisoning C.Thalassemia D.Sicklecellanaemia E.Acutelymphoblasticleukaemia

Answer:c)thalassemia. Thispatientislikelytohavethalassemiatrait(probablyalphathalassemia).Thosewith alpha thalassemia trait are clinically normal, but their hemoglobin is slightly low and theirhemogramdemonstratesmicrocyticindices.

Dr.KhalidYusufElzohrySohagTeachingHospital2012 110

www w.MRCPass.c com

Haem matology

Hae ematologyQ Q110: A50 0yearoldw womanhas alifelong historyofa anemia.Her rmotherha asasimilarhistory. Laboratoryvalu uesshow emoglobin1 10.5g/dl Hae Hae ematocrit33 3 MCV V66 Bloo film: microcytic, no od ormochrom red cells A fewelli mic s. iptocytes an target cells are nd c note ed. ichofthefo ollowingist themostlik kelyconditio on? Whi
A.Irond deficiencyan naemia B.Hered ditarysphero ocytosis C.Thalassemia D.G6PD Ddeciency E.Autoimmunehaemolyticanae emia

Answer:c)Thalassemia. This spatientha aseithertha alassemiat traitorbeta athalassem miaminor.M Mutationsin nglobin genes cause thalassemia Alpha thalassemia affects th alphaglo t as. he obin gene(s Beta s). thal lassemiaaff fectsoneorbothofth hebetaglobingenes. Inbetathalassemiamajor(ie, hom mozygous b beta thalas ssemia), th product he tion of betaglobin c chains is s severely imp paired,beca ausebothb betaglobin genesarem mutated.In nbetathala assemiamin nor,one ofthebetaglo obinchainsi isimpaired. . The severe anemia result e ting from this disease, if untreat ted, can res in high sult houtput card diacfailure,whichcaus sesthehigh hestmortali ity.

Bloo odfilmshow wingtargetcells(low erleftand upperrightconneras swellaselli iptocyte inth helowerlef ft)

Dr.K KhalidYusuf fElzohrySo ohagTeachin ngHospital 2012 111

www w.MRCPass.c com

Haem matology

Hae ematologyQ Q111: A48 8yearoldm malewasreferredforinvestigation nofanemia a. Reu ultsshow: hem moglobinof7.2g/dL awhitebloodc cellcounto of3x10^9/l l teletcountof60x10^9 9/l plat Onphysicalexa am,hissple eenwasenlarged. ipheral blood showed teardrops and a leukoerythroblastic sme d s ear. Bone marrow Peri biop psyshowed dincreasednumbersof fmegakaryo ocytesandgradeIIIfib brosis. Wha atisthedia agnosis?
A.Folate edeficiencyanaemia B.Coom mbspositivehaemolyticanaemia C.Myelo ofibrosis D.Multi iplemyeloma E.Chron nicmyeloidle eukaemia

Answer:c)mye elofibrosis. In m myelofibros splenom sis, megaly occu with a fibrotic pro urs ocess. Haem molytic ana aemia is nottypicallysteroidrespo onsive. Leuc coerythrobl lasticanaem mia(redce ellandwhitecellprecu ursors)are seenontheblood film Bone pain, bleeding (platelet d m. g dysfunction may occu but are not charac n) ur cteristic. Fibr roustissuei infiltrationo ofthebone emakesitd difficulttoaspiratebon nemarrow.

Mar rrowbiopsy ydemonstra atingsigificantfibrosis

Dr.K KhalidYusuf fElzohrySo ohagTeachin ngHospital 2012 112

www.MRCPass.com

Haematology

HaematologyQ112: A 16 yearold boy with sickle cell disease complains of acute breathlessness. He has a respiratoryrateof35breathsperminute.O2satura onsare75%onroomairand85% on 100% oxygen. There is reduced air entry bilaterally, but no added sounds. Investigationsshow: pO26.2kPa CXR:bilateralbasalinfiltrates Hb7.5g/dl WCC14x10^9/l platelets200x10^9/l Themostappropriatemanagementis:
A.IVantibiotics B.Noninvasiveventilation C.IVfluids D.Bloodtransfusion E.Urgentexchangetransfusion

Answer:e)Urgentexchangetransfusion. The diagnosis is acute chest syndrome. The acute chest syndrome (ACS) in sickle cell disease(SCD)canbedefinedas: 1.anewinfiltrateonchestxray 2. associated with one or more NEW symptoms: fever, cough, sputum production, dyspnea,orhypoxia. Exchange blood transfusions are indicated in cases of cerebrovascular accidents and acutechestsyndrome. Theyareperformedoccasionallyinpatientswithacutesequestrationcrisisorincasesof priapism that do not resolve after adequate hydration and analgesia . Exchange transfusion consists of replacing the patient's RBCs by normal donor RBCs, decreasing HbStolessthan30%.

Dr.KhalidYusufElzohrySohagTeachingHospital2012 113

www.MRCPass.com

Haematology

HaematologyQ113: A35yearoldpa enthasschizophrenia.Heisonclozapine,temazepamandamoxycillin forarecentchestinfection. Heisadmittedunwellthefollowingbloodresults: Hb 2.0 g/dl, WCC 2 x 10^9/l, Neutrophils 0.3 x 10^9/l, platelets 180 x 10^9/l, urea 6 mol/l,crea nine80mol/l, sodium140mmol/l,potassium3.8mmol/l. Whichoftheseislikelytobethecauseofneutropenia?
A.Clozapine B.Amoxycillin C.Myelofibrosis D.Myeloma E.Sepsis

Answer:a)clozapine. Clozapineisassociatedwithneutropeniaandagranulocytosisaremainhaematological complications.SevereneutropeniacanbetreatedwithGCSF.

Dr.KhalidYusufElzohrySohagTeachingHospital2012 114

www w.MRCPass.c com

Haem matology

Hae ematologyQ Q114: A 30 year old male prese ents with p painless cer rvical and a axillary lymphadenopa athy. He also ocomplains soffeveran ndpruritus.Lymphno odebiopsyd demonstrat tesReedSternberg cells s. ichfeaturewouldindic catetheworstprognos sis? Whi
A.Sweating B.Inguin nallymphadenopathy C.Bonemarrowinvo olvement D.Prurit tus E.Fever r

Answer:c)Bon nemarrowinvolvement. The epatientha asHodgkins sdisease.In nvolvement tofthebonemarrow wouldclas ssifythe pati ientasstageIV(modifiedAnnArb borclassification)indic catingpoorprognosis. *St tageIis inv volvemento ofasingle lymphnoderegion(I) )orsinglee extralympha site atic (Ie) * St tage II is involvement of twoor more lymph node reg gions on the same side the e eof diap phragm(II)orofonely ymphnoderegionandacontiguousextralym mphaticsite(IIe) *St tageIIIisinvolvement oflymphn noderegionsonboths sidesofthe ediaphragm m,which may yincludeth hespleen(IIIs)and/orlimitedcont tiguousextr ralymphatic corganors site(IIIe, IIIes s) *St tageIVisdis sseminated dinvolveme entofoneormoreextr ralymphatic corgans

Ree edSternberg gcellshowingprominentnucleol li

Dr.K KhalidYusuf fElzohrySo ohagTeachin ngHospital 2012 115

www w.MRCPass.c com

Haem matology

Hae ematologyQ Q115: A25 5yearoldladyhasam motherwho ohashada asplenectom myforanae emia.Shep presents unw with ab well bdominal p pain and vo omiting. Examination reveals a t tender right upper quadrantandjaundice,as swellas4cmsplenomegaly. Herbloodssho ow: Hb9 MCV95 9.0g/dl 5 MCHC33g/dl( (3235) WCC11 1x10^9/l telets200x10^9/l re culo ocytes180x x10^9/l(50 0100) plat urea a5.5mol/ /l crea nine65mo ol/l sodium137mm mol/l mol/l potassium4.2mm bilir AST60U/l rubin48m mol/l albumin38g/l ALP P450U/l LDH H650U/l(10250) ichoneofth hefollowing gisthelikelydiagnosis s? Whi
A.Autoi immunehae emolyticanae emia B.Pancr reatitis C.Hered ditarysphero ocytosis D.Sickle ecelldisease e E.Acute emyeloidleu ukaemia

Answer:c)here editarysphe erocytosis. Her reditarysph herocytosisismostlikelydueto thefamily historyand dpresentati ion.The bloo tests sh od how likely haemolysis The hist s. tory also s suggests ch holecystitis due to galls stones. Gall lstonesoccu urinpatien ntswithrecu urrenthaem molysis(pigmentedsto ones).

Spherocytesar resmall,rou underythro ocytesthatl lackcentral lpallor(arro ows)


Dr.K KhalidYusuf fElzohrySo ohagTeachin ngHospital 2012 116

www.MRCPass.com

Haematology

HaematologyQ116: A22yearoldwomanpresentswithanacutepulmonaryembolisminthe8thweekof pregnancy. Whatisthemostappropriatetreatmentforthispatientthroughoutherpregnancy?


A.Clopidogrel B.Intravenousheparin C.Subcutaneouslowmolecularweightheparin D.Dipyridamole E.Warfarin

Answer:C)Subcutaneouslowmolecularweightheparin. Warfarinshouldonlybeusedinthethirdtrimester.Itisteratogenicanduseinthefirst 2trimestersarenotrecommended.Atpresent,thepa entshouldhaveLMWH.

Dr.KhalidYusufElzohrySohagTeachingHospital2012 117

www.MRCPass.com

Haematology

HaematologyQ117: A 45 year old woman has von Willebrand's disease. She is going to undergo tooth extraction. Which one of the following is the best management option in mild von Willebrand's diseasepriortosurgery?
A.DDAVP B.FactorVIIIconcentrate C.Freshfrozenplasma D.FactorIXconcentrate E.Bloodtransfusion

Answer:a)DDAVP. Outofallthechoices,DDAVPisthemostpragmaticoption.Freshfrozenplasmaorvon Willebrandfactorcanbeusedincasesofseverebleedingbutshouldnotbeusedinmild cases.

Dr.KhalidYusufElzohrySohagTeachingHospital2012 118

www w.MRCPass.c com

Haem matology

Hae ematologyQ Q118: A80 0yearoldw womanpres sentswithg generalisedabdominalpains. inve estigationss show: WB Hb1 10.3g/dl BC17x10^9/l plts80x10^9/ /l MCV85 odfilmshowsnucleate edredcells s,smallnum mbersofpr romyelocyte es,myelocy ytesand Bloo met tamyelocyte es atisthemo ostlikelycau useofthese ehaematolo ogyresults? ? Wha
A.Folate edeficiency B.Sickle ecelldisease e C.Osteo oporosis D.Thala assaemiamin nor E.Myelo ofibrosis

Answer:e)Mye elofibrosis. Bloo odfilmfind dingsdescribealeucoe erythroblas sticbloodp picturewhic chischarac cterized by granulocyte and eryt e throid prec cursors in t the peripheral blood. Common causes include: myelofibr rosis bonemar rrowinfiltra ationwithle eukaemia severemegaloblastic canaemia sicklecell lcrisis thalassae emiamajor osteopetrosis

Ale eucoerythro oblasticpict turepresenceofimm maturemyel loidandnucleatedred dcellsin theperipheralblood

Dr.K KhalidYusuf fElzohrySo ohagTeachin ngHospital 2012 119

www.MRCPass.com

Haematology

HaematologyQ119: A9yearoldboyisunwellhavingingestedabottleofdye.Onexamination,heisafebrile but has tachypnea, cyanosis, and drow siness. He is given 100% oxygen but does not improve. Whatisthemostlikelydiagnosis?
A.Methaemoglobinaemia B.Sicklecellanaemia C.Thalassemia D.Congenitalcyanoticheartdisease E.Henochschonleinpurpura

Answer:a)methaemoglobinaemia. Cyanosis that is unresponsive to oxygen therapy is most likely due to methemoglobinemia. Methaemoglobinaemia is a cause of cyanosis because it causes the formation of reducedHb>1.5g/dl.ItisduetooxidisedironfromFe2+toFe3+inHbandmaycause precipitationasHeinzbodies. Chemicals which are oxidising agents may cause this e.g. aniline dyes, chlorates, nitrates, nitrophenols, primaquine and sulphonamides. Treatment is with methylene blueifmethaemoglobin>3.0g/dL.

Dr.KhalidYusufElzohrySohagTeachingHospital2012 120

www.MRCPass.com

Haematology

HaematologyQ120: An 75 year old woman is admitted for routine surgery. She is found to have a haemoglobinof7.8g/dLwithhypochromic,microcyticindicesandthebloodfilmshows pencilcells. Whatisthemostappropriatemanagement?
A.Transfuseblood B.Continuewithsurgery C.Sendhaematinicsandtreatwithferroussulphate D.Sendhaema nicsandtreatwithvitaminB12 E.Requestabonemarrowexaminationtoexcludemyelodysplasia

Answer:c)Sendhaematinicsandtreatwithferroussulphate. Iron deficiency is a common cause of anaemia, especially in the elderly. In this age group,itisoftenduetopoordietaryintake,althoughOGDorcolonoscopymayneedto bedonetoexcludeGIbleedingasacauseofbloodloss.

Dr.KhalidYusufElzohrySohagTeachingHospital2012 121

www.MRCPass.com

Haematology

HaematologyQ121: A 12 year old boy presents with breathlessness and cough. On examination he ispale andjaundiced.HisHbis5.5g/dlandperipheralsmearshows50%sickledcellswith<1% reticulocytes. Whichofthefollowingisresponsibleforhiscondition?
A.SalmonellaInfection B.Pneumococcalinfection C.H.Influenzaeinfection D.Cytomegalovirusinfection E.Parvovirusinfection

Answer:e)Parvovirusinfection. The child has sickle cell anaemia with an aplastic crisis. This is most commonly precipitatedbyparvovirusB19.Salmonellacancauseosteomyeli sandHinuenzaecan causepneumoniainpatientswithsicklecelldisease.

Dr.KhalidYusufElzohrySohagTeachingHospital2012 122

www.MRCPass.com

Haematology

HaematologyQ122: A 48 year old man being has polyuria, polydipsia and impotence. On examination, he hasapalpableenlargedliver. Investigationsshow: Alanineaminotransferase80U/L(535) Aspartateaminotransferase92U/L(131) Albumin36g/lFas ngplasmaglucose7.4(3.06.0) Ferri n800ug/L(15300) Whichoneoffollowingisthenextbestinvestigation?
A.Oralglucosetolerancetest B.Serumtransferrinreceptors C.Liverbiopsy D.Transferrinsaturation E.Bonemarrowbiopsy

Answer:d)transferrinsaturation. The diagnosis is haemochromatosis suggested by high ferritin. The best test now is transferrinsaturation.Ifthisishigh,HFEgeneanalysisshouldalsobeperformed.

Dr.KhalidYusufElzohrySohagTeachingHospital2012 123

www w.MRCPass.c com

Haem matology

Hae ematologyQ Q123: A65 5yearoldw womanpre esentswith a6monthhistoryof backpain. Shealsom men ons poly yuriaandle ethargy. Inve estigationsr reveal: haemoglobin9 9.2g/dL(11.516.5) cellcount3. .5xl0^9/L(411) whitec teletcount275x10^9/L(15040 00) totalpr rotein85g/ /L(6176) plat albu umin32g/L L(3749) urea18 8mmol/l(37) crea nine350micromol/L a calcium m2.85mmo ol/l(2.252.7 7) L(60110) plas smaglucose e5.5(3.06 6.0) urin nedipsticka analysispro otein+blood d+ rena alultrasoun ndnormals sizedkidney ys ichoneofth hefollowing ginvestigat tionsisappropriate? Whi
A.Liverultrasound B.Throm mbophiliascreen C.Serum melectropho oresis D.HAMtest E.Throm mbintime

Answer:c)seru umelectrop phoresis. Ana aemia, bone pain and hypercalca e d aemia are typical pre esenting fea atures of multiple m mye eloma.Seru umelectrop phoresiswil llshowam monoclonalb bandofeith herIgG,IgM MorIgA variety.BenceJonesprote einmayalso obefoundintheurine e.About15 5%ofpa en ntshave BJp proteininth heirurinew withoutaparaproteinae emia.

Plas smacellsinmultiplemyeloma

Dr.K KhalidYusuf fElzohrySo ohagTeachin ngHospital 2012 124

www w.MRCPass.c com

Haem matology

Hae ematologyQ Q124: A 1 year old female with known sickle cell anaemia h been unwell with fevers, 13 has ano orexiaandse everepains sinthemus sclesandjointsofheru upperandlowerlimbs s. Herbloodtests sshow: Hb4 WC 4g/dl CC3.9x10^9/l Plt7 75x10^9/l Abs solutere cu ulocytecount0.3(0.51 1.5) LDH H280(852 285)IU/L Bilir rubin15(1 22)mol/l Bloo odfilmreve ealssicklece ellswithab bsenceofpo olychromasia. atisthelike elyscenario o? Wha
A.Aplas sticsicklecris sisduetopa arvovirus B.Acute ehaemolysis s C.Splenicsequestration D.Sickle echestsyndr rome E.Folate edeficiency

Answer:a)apla asticsicklec crisisdueto oparvovirus s. The most likel cause is aplastic cr e ly risis, the co ommonest cause bein parvovir ng rus. The retic culocyteco ountislow,bilirubinisnormaland dLDHisnor rmal. Poly ychromasia isthepres senceofgre eycoloured dredcellso onfilm,indi icatingpres senceof incr reasedretic culocytes. The elackofpoly ychromasia aonthebloodfilmsuggestsaplast ticanaemia a.

Sick Cell an kle naemia Ap plastic Crisi Numero is. ous sickled RBC's ar present (small d re t arro ows)and A single nuc cleated RBC is noted (large arr C row ). Not the abse te ence of poly ychromasia. Hae ematologyQ Q125:
Dr.K KhalidYusuf fElzohrySo ohagTeachin ngHospital 2012 125

www.MRCPass.com

Haematology

A 70 year old man is admi ed for inves ga on of jaundice and anorexia for several weeks.6weeksago,hehadbeenprescribedatwoweekcourseofAugmentinbyhisGP forasevereupperrespiratorytractinfectionandwasalsotakingIbuprofenforgout.He livesaloneandhasnothadrecenttravel.Hedrinks2unitsofalcoholaweek. Investigationsreveal: Albumin40g/L Bilirubin255umol/L(122) AST260iu/L(535) AlkalinePhosphatase220iu/l(50110) Abdominalultrasoundrevealsgallsonesbutnoevidenceofcholecystitis. Whatisthemostlikelycauseofthejaundice?
A.Allopurinol B.Augmentin C.Alcoholicliverdisease D.Viralhepatitis E.Ibuprofen

Answer:B)Augmentin. Thebloodresultsareconsistentwithcholestaticjaundice.Coamoxiclav(augmentin)is awellknowncauseofthis.Thereisoftenalatencytimebetweentheuseofdrugand onsetofcholestaticjaundice.Studiesofcasereportsshowedthattheonsetofjaundice istypicallyfrom2weeksto6weeksforpenicillins. Some drugs cause a reaction even later several months. How ever, one should scrutinisethedatainyourexamquestionbecausethestructuremaybedifferent.

Dr.KhalidYusufElzohrySohagTeachingHospital2012 126

www.MRCPass.com

Haematology

HaematologyQ126: A27yearoldwomanhasroutinebloodtestsduringherpregnancy. Hb9.8g/dl WBC5.4x10^9/l Plts260x10^9/l MCV69 MCH17.2 Whatisthemostlikelydiagnosis?


A.Folatedeficiency B.Betathalassemiatrait C.B12deciency D.Sideroblasticanaemia E.Anaemiaofchronicdisease

Answer:b)Betathalassemiatrait. Iron deficiency anaemia and thalassemia trait are the two most likely diagnoses of microcytic anaemia. Iron deficiency is best diagnosed by a low ferritin level. Beta thalassemia trait is diagnosed by the presence of a raised HbA2 (with Hb electrophoresis).

Dr.KhalidYusufElzohrySohagTeachingHospital2012 127

www.MRCPass.com

Haematology

HaematologyQ127: A42yearoldwomanwithalonghistoryofdrinkingalcoholhasepilepsy.Shehasbeen on phenytoin and carbamazepine since the diagnosis was made 5 years ago. Investigationsreveal: Haemoglobin9.5g/dL(1316) MCV118fL(8096) whitecellcount2.5x10^9/L(411) plateletcount72x10^9/L(150400) Whatisthemostlikelyexplanationfortheseresults?
A.Myelodysplasia B.Aplasticanaemia C.Folicaciddeficiency D.Sideeffectofcarbamazepine E.Chroniclymphocyticleukaemia

Answer:C)Folicaciddeficiency. Folic aciddeficiencywouldfittheclinicaldescriptionandisaknownadverseeffectof longtermphenytointherapy.

Dr.KhalidYusufElzohrySohagTeachingHospital2012 128

www.MRCPass.com

Haematology

HaematologyQ128: A40yearoldladywhohasbeenonwarfarinfordeepvenousthrombosispresentswith upperGIbleeding.HerINRwas9.1. Whatistheappropriatetreatment?


A.Protamineconcentrate B.Platelettransfusion C.Cryoprecipitate D.FactorVIIItransfusion E.Tranexemicacid

Answer:C)cryoprecipitate Fresh frozen plasma and cryoprecipitate are employed in treating patients with coagulopathiesduetodeficiencyofoneormorecoagulationfactors.Theseconditions mayoccurduetoacceleratedconsumptionofcoagulationfactors,(e.g.bleeding,DIC), impaired factor production states (vitamin K deficiency, w arfarin effect, liver disease, congenitalfactordeficiencies). Cryoprecipitatecontainsfibrinogen,FactorVIII,vonWillebrandsFactor,FactorXIII,and fibronectin.

Dr.KhalidYusufElzohrySohagTeachingHospital2012 129

www.MRCPass.com

Haematology

HaematologyQ129: A65yearoldwomanpresentswithlethargy. She has a Hb of 9.0 g/dl, WCC of 12 x 10^9/l, platelets of 100 x 10^9/dl, blood lm showsspherocytes,polychromasiaandsmearcells.DirectCoomb'stestispositive. Whichisthelikelycauseoftheanaemia?
A.Thromboticthrombocytopenicpurpura B.Autoimmunehaemolyticanaemia C.Idiopathicthrombocytopenicpurpura D.Hereditaryspherocytosis E.B12deciency

Answer:b)autoimmunehaemolyticanaemia. The blood film and positive Coomb's test points tow ards autoimmune haemolytic anaemia.ThismayberelatedtoaleukaemicprocessinthepatientwithalsoraisedWCC andthrombocytopenia

Dr.KhalidYusufElzohrySohagTeachingHospital2012 130

www.MRCPass.com

Haematology

HaematologyQ130: An18yearoldgirlisbeinginves gatedforworseningmenorrhagiaandgumbleeding. Sheundergoesaseriesofbloodtestswhichareshownbelow: Hemoglobin12.3g/dl(10.513.5) WBC7.6x10^9/L(6.017.5) Platelets328x10^9/L(156369) APTT52.6s(28.038.0) BleedingTime71/2minutes(<5minutes) ProthombinTime11.6s(10.012.8) ThrombinTime17.3s(16.022.0) FactorVIII0.18U/ml(0.601.50) FactorIX0.92U/ml(0.601.50) vWFAg0.16s(0.781.53) VWFristoce ncofactor<0.10U/ml(0.501.50) Whichofthefollowingisthelikelydiagnosis?
A.VonWillebrandsdisease B.FactorVleiden C.CarrierforHaemophiliaA D.Acutemyeloidleukaemia E.Idiopathicthrombocytopenia

Answer:a)VonWillebrandsdisease. Sheismostlikelytohavetype1VonWillebranddisease(vWD),wheretheprolongation oftheAPTTisduealowfactorVIIIlevelwhichoccurssecondarytothelowVWFlevel. Von Willebrand disease (VWD) is a group of genetically heterogenous disorders resulting in abnormal function of the Von Willebrand factor (VWF). More than 100 mutationshavebeendescribed.Symptomsincludemucocutaneousbleeding(epistaxis, easybruising,prolongedbleedingafterminortrauma,menorrhagiaandgastrointestinal bleeding)ofvaryingseverity.Hemarthrosisisrelativelyuncommon.Unlikehemophilia, the mode of inheritance is predominantly autosomal dominant (some autosomal recessivevariantshavebeendescribed). Type 1 vWD is characterized by a par al quan ta ve decrease of qualita vely normal vWFandFVIII. Type2AvWDisinheritedischaracterizedbynormaltoreducedplasmalevelsoffactor VIIIc(FVIIIc)andvWF. Type2BvWDischaracterizedbyareduc oninthepropor onofhighmolecularweight vWFmultimers,whiletheproportionoflowmolecularweightfragmentsareincreased.

Dr.KhalidYusufElzohrySohagTeachingHospital2012 131

www.MRCPass.com

Haematology

HaematologyQ131: A5year oldboywas brought totheemergencydepartment byhis motherforoozing blood from his mouth following a fall. His mother said that he tended to bleed for prolongedperiodsfromhisimmunizationsites,buttherewasnohistoryofbruisingor hematomas.Thepatientwasonantibioticsforarecentearinfection.Therewasafamily historyofsimilarbleedinghissisterandmotherbeingaffected. Bloodtestsshow: Hemoglobin13.3g/dl(10.513.5) Hematocrit35.4%(33.039.0) WBC6.9x10^9/L(6.017.5) Platelets350x10^9/L(156369) PT12.3s(10.012.8) APTT38.2s(24.433.2) Bleeding me12minutes(29) Whatisthediagnosis?
A.HaemophiliaA B.HaemophiliaB C.Vonwillebrand'sdisease D.Acutelymphoblasticleukaemia E.Acutemyeloidleukaemia

Answer:c)vonwillebrand'sdisease. In this case, the family history and also prolonged bleeding time suggests von Willebrand's disease. Von Willebrand's disease has mostly autosomal dominant inheritance. Symptoms of von willebrand's disease include mucocutaneous bleeding (epistaxis, easy bruising, prolonged bleeding after minor trauma, menorrhagia and gastrointestinal bleeding) of varying severity. The quantitative assay (VWF AG) and functionalassay(VWFristocetincofactor/collagenbindingcapacity)arerecommended for diagnostic purposes. Approximately 25% of patients with type 1 VWD have aPTT resultsoutsideofthereferencerange. DDAVPcanraisethelevelsofvWFintheblood. FactorVIIIconcentratesandplasmaproductscanbeused. Usingtheearlobemethod,anormalbleeding meisbetween1and4minutes.Using theforearmmethod,anormalbleeding meisbetween2and9minutes.

Dr.KhalidYusufElzohrySohagTeachingHospital2012 132

www.MRCPass.com

Haematology

HaematologyQ132: A75yearoldladyhaspresentedwithsymptomsconsistentwithaUTI.Herbloodtests showHbof10.0g/dl WCCof45x10^9/l platelets160x10^9/l neutrophilcountis12x10^9/l(1.57) lymphocytecountis27x10^9(1.54) Whichofthefollowingtestsisthebesttoelucidateadiagnosis?


A.Bonemarrowtrephine B.Whitecellimmunophenotyping C.Hbelectrophoresis D.Ultrasoundofabdomen E.Splenicbiopsy

Answer:b)whitecellimmunophenotyping. Ahighwhitecellcountwithpredominantlymphocytosisandanaemia suggests a possible leukaemia such as Chronic lymphocytic leukaemia. Immunophenotyping can be used for classification of undifferentiated leukemia as lymphoidormyeloidandsubclassificationofleukemias.

Dr.KhalidYusufElzohrySohagTeachingHospital2012 133

www w.MRCPass.c com

Haem matology

Hae ematologyQ Q133: A 2 year old woman a 28 d attends A+E with a history of decreased consciousne E c ess. Her inve estigationss show: Hb6 6.7g/dl WBC7.8x10^9 9/l /l plts15x10^9/ TT34secs APT PT1 16secs Fibr rinogen1.6g/dl Crea nine180mol/l a Bloo odfilm:redcellfragme entation,po olychromasiaandBurrcells. atisthemo ostlikelydia agnosis? Wha
A.Multi iplemyeloma B.Throm mboticthrom mbocytopeni iapurpura C.Sever reirondeficiency D.Disseminatedintr ravascularco oagulation E.Acute elymphoblas sticleukaemia

Answer:b)Thromboticthrombocytopeniapurpu ura. Thro ombotic thrombocy ytopenic purpura is a syndrome characterized by microangiopathic hemoly ytic anemia, thromb bocytopenia neurolog abnorm a, gic malities, feve er,andrena aldysfuncti ion.Aspect trumofpre esentations relatedto thrombotic cevents can occur,alteredconscio ousness,sei izures,fever,myalgiaa andarthralg giaoccur.W Withthe introduction o plasma exchange ( of (recommen nded treatm ment), the survival ra has ate imp provedfrom mapproxima ately3%pri iortothe19 960sto82% %.

The blood film in microang e giopathic haemolytic c schi istocytes(fra agmentedc cells)andBurrcells.
Dr.K KhalidYusuf fElzohrySo ohagTeachin ngHospital 2012

anaemia a

demonstrating

134

www.MRCPass.com

Haematology

HaematologyQ134: A30yearoldmanpresentswithpainlesslumpsintheneck.Thishasbeenpresentfor the last 7 weeks. He has lost about 10 kgs in weight over the last six months and complainsoffeverwithnightsweats.Hesmokes20cigarettesaday. Onexaminationthereareseveralenlargedlymphnodesintheleftsupraclavicularfossa. Investigationsareasfollows: Hb10.3g/dL MCV85 WBC16.0x10^9/L Neutrophils55% Lymphocytes34% ESR57mm/hour Whichofthefollowingtestswouldbemostappropriatetoconfirmthediagnosis?
A.Chestxray B.Kveimtest C.SputumforAFB D.Lymphnodebiopsy E.Ultrasoundscanofabdomen

Answer:d)lymphnodebiopsy. The most likely diagnosis in a patient who has cervical lymphadenopathy and B symptomsislymphoma.

Dr.KhalidYusufElzohrySohagTeachingHospital2012 135

www.MRCPass.com

Haematology

HaematologyQ135: A 31 year old white woman has recurrent episodes of epistaxis. Physical examination revealedtelangiectasiasonherforeheadandbuccalsurfaceoftheoralmucosa. Whatisthelikelydiagnosis?
A.Wegener'sgranulomatosis B.Goodpasture'ssyndrome C.OslerRenduWebersyndrome D.HaemophiliaA E.VonWillebrand'sdisease

Answer:c)OslerRenduWebersyndrome. The diagnosis is hereditary haemorrhagic telangiectasia (Osler Rendu Weber syndrome). Multiple telangiectasia are usually seen on the hands and around the mouth.Arteriovenousmalformationsareassociated(pulmonaryorcranial).

Dr.KhalidYusufElzohrySohagTeachingHospital2012 136

www.MRCPass.com

Haematology

HaematologyQ136: A50yearoldmanhasbronzepigmenta on.Hehasafamilyhistoryofliverproblems. Clinicalexaminationrevealshepatomegaly.Hisinvestigationsshow: Hb14.0g/dl MCV90 MCHC30g/dl(3235) WCC8x10^9/l platelets180x10^9/l PT17s(11.515.5) APTT35s(2438) urea5mol/l crea nine80mol/l sodium140mmol/l potassium3.6mmol/l bilirubin26mol/l AST70U/l ALP140U/l albumin32g/l iron50mol/l(1429) ferri n650g/l(15200) transferrinsatura on80% Whichofthefollowingisthelikelydiagnosis?
A.Alcoholicliverdisease B.Haemochromatosis C.Addison'sdisease D.ChronichepatitisC E.Porphyria

Answer:b)haemochromatosis. Thereishighironandferri n.Transferrinsatura onof>50%ishigh.Thisislikelytobe haemochromatosis,whichisautosomalrecessive.Venesectioncanhelpreduceferritin levels(aimingfor50g/l).

Dr.KhalidYusufElzohrySohagTeachingHospital2012 137

www w.MRCPass.c com

Haem matology

Hae ematologyQ Q137: A 3 year old man has had 2 epis 30 sodes of ha aemetemes followin consump sis ng ption of seve eralpintsof fbeer. Inve ga onsreveal:Hb6 es 6g/dl,WCC C1.7x10^9 9/l,platelets s4x10^9/l l,PT19s,AP PTT52s, brinogen0.3g g/l(25),fib brindegrada onprodu a ucts120ug/ /ml(<10). odfilmreve ealspredom minantlyblastcellscontainingAue erRods. Bloo ichofthefo ollowingist themostlik kelydiagnos sis? Whi
A.Aplas sticanaemia B.Disseminatedintr ravascularco oagulation C.Immu unethromob bocytopenicpurpura D.Acute epromyelocy yticleukaem mia E.Acute elymphoblas sticleukaemia

Answer:D)acu utepromyel locyticleuka aemia. Acu utemyeloid dleukaemia aisdenedas20%or moremyeloblastsint thebonem marrow. Acute promyelocytic leuk kaemia (M3 is charac 3) cterized by presence of promye y elocytes. The ereisanassociationwit ththecytog genetictran nsloca ont t(15:17).

Aue errodsaree elongated,bluishredr rodscompo osedoffuse edlysosoma algranules,seenin thecytoplasmofmyelobla asts.

Dr.K KhalidYusuf fElzohrySo ohagTeachin ngHospital 2012 138

www.MRCPass.com

Haematology

HaematologyQ138: A62yearoldwomanhadsuccessfulkneesurgery.Aweekfollowingsurgery,shehada DVTdespiteprophylacticdosesoflowmolecularweightheparin. Herbloodtestsshowed:INR1.1,APPT37s,Fibrinogen4.6g/l,Hb12.8g/dl,platelets18 x10^9/l,WCC22x10^9/l.Bloodlmshowedanisocytosis. Whatisthelikelydiagnosis?


A.Idiopathicthrombocytopenicpurpura B.Disseminatedintravascularcoagulation C.Thromboticthrombocytopenicpurpura D.Heparininducedthrombocytopeniaandthrombosis E.Haemolyticuraemicsyndrome

Answer:d)Heparininducedthrombocytopeniaandthrombosis. The ming of events, about 7 days a er commencing heparin suggests an immune mediated phenomenon. Despite thrombocytopenia the patient is predisposed to thrombosis. Platelet aggregation results in thromboembolic events The normal fibrinogensuggestsDICisunlikely.

Dr.KhalidYusufElzohrySohagTeachingHospital2012 139

www.MRCPass.com

Haematology

HaematologyQ139: A 35 year old lady has had treatment with penicillamine for rheumatoid arthritis. She presentswithweaknessandpallorclaimingthatitmaybeasideeffectofthedrug. HerHbis5g/dl,WCCis2x10^9/l,plateletcountis15x10^9/l,INRis1.0andAPTTis 27s. Whichofthefollowingisthemostlikelydiagnosis?
A.Acutemyeloidleukaemia B.Myelodysplasia C.Aplasticanaemia D.Folatedeficiency E.B12deciency

Answer:c)aplasticanaemia. Thereispancytopeniaduetobonemarrowfailure.Aplasticanaemiacanbecongenital (Fanconi'sanaemia)oracquiredduetodrugs(benzenecompounds,insecticides,goldor penicillamine). Treatmentiswithantilymphocyteglobulin,cyclosporinormethylprednisolone.

Dr.KhalidYusufElzohrySohagTeachingHospital2012 140

www.MRCPass.com

Haematology

HaematologyQ140: An8yearoldboyisbeinginves gatedforshortstature.Examina onrevealed2cafau laitspots.Bloodtestsshowed:Hb9g/dl,WCC2.5x10^9/l,platelets28x10^9/l. Whatisthelikelycauseofanaemia?


A.Irondeficiency B.Acutelymphoblasticleukaemia C.Fanconisanaemia D.Folatedeficiency E.Multiplemyeloma

Answer:c)Fanconisanaemia. Fanconis anaemia o en presents at age < 10 with growth retardation, renal defects andcafaulaitspots.Inheritanceisautosomalrecessive.10%ofpa entsmaydevelop acutemyeloidleukaemiawithtime.

Dr.KhalidYusufElzohrySohagTeachingHospital2012 141

www.MRCPass.com

Haematology

HaematologyQ141: A35yearoldwomanpresentswithjaundiceandlethargy. Herinvestigationsreveal: Haemoglobin8.0g/dL reticulocytecount150x10^9/L(2585) serumbilirubin75umol/L Herbloodfilmrevealspresenceofspherocytes Whichofthefollowingisthenextusefulinvestigation?


A.Endoscopy B.Glucose6phosphatedehydrogenaseac vity C.Directantiglobulintest D.Redcellosmoticfragility E.Haemoglobinelectrophoresis

Answer:c)directantiglobulintest. Thedirectantiglobulintest(DAT)isusedtodetectIgGorC3boundtothesurfaceofthe redcell.Inpatientswithhemolysis,theDATisusefulindeterminingwhetherthereisan immuneetiology. Nonimmune causes of hemolysis such as DIC, thrombotic thrombocytopenic purpura, mechanical hemolysis such as those due to artificial valves or burns, hemoglobinopathies (sickle cell, thalassemia), red cell enzyme deficiencies (G6PDP, pyruvate kinase), and red cell membrane defects (hereditary spherocytosis, PNH) will haveanegativeDAT. Immune causes of hemolysis including autoimmune hemolytic anemias, drug induced hemolysis,anddelayedoracutehemolytictransfusionreactionsarecharacterizedbya positiveDAT.

Dr.KhalidYusufElzohrySohagTeachingHospital2012 142

www.MRCPass.com

Haematology

HaematologyQ142: A 40 year old man has had a bow el opera on. 48 hours later he becomes febrile, hypotensiveandunwell.Hisinvestigationsshow: Hb12.6g/dl WBC17.4x10^9/l plts45x10^9/l Ddimer16,000(<500)ng/dl Fibrinogen82(180363)mg/dl Haptoglobin6(16200)mg/dl INR2.4(1) APTT50(<34) Whatisthemostlikelycauseofthethrombocytopenia?
A.Immunethrombocytopenia B.Disseminatedintravascularcoagulation C.Heparininducedthrombocytopenia D.Thromboticthrombocytopenicpurpura E.Aplasticanaemia

Answer:b)Disseminatedintravascularcoagulation. Disseminated intravascular coagulation is caused by inappropriate and excessive ac va onofthehaemosta csystems.60%arecausedbyGramnegativesepsis. Othercausesincludeviralinfections,metastaticcarcinoma,leukaemia,obstetriccauses, extensivetraumaandburns. APTT, PT (INR) and TT are all prolonged, platelets and fibrinogen are low , D dimers/FDPs are elevated. Other presenting laboratory abnormalities include uremia, elevated creatinine, elevated lactate dehydrogenase, decreased haptoglobin, bilirubinemia and lactic acidosis. Schistocytes usually are evident on peripheral smear. Treatmentisofunderlyingcausesandbycontrolofthe haemorrhagicstate.Platelets, blood,cryoprecipitateandfreshfrozenplasmamayallberequired.

Dr.KhalidYusufElzohrySohagTeachingHospital2012 143

www w.MRCPass.c com

Haem matology

Hae ematologyQ Q143: A30 0yearold manpresen ntswithpainlessenlar rgementof fhiscervica allymphno odes.He also ocomplains soffeveran ndnightsw weats.Heh haslost1stoneinweig ghtoverthe epast3 mon nths.Chestxrayshowsmediastin nalwidening g. A ly ymph node biopsy is performed and this reveals a background of lymph e d d hocytes, plas sma cells, h histiocytes, eosinophils neutroph and fibr s, hils roblasts. Sca attered wit thinthis backgroundinf filtrateare anumber oflargecellswithtwo olargenucleiwithpro ominent nucleoli. is: Thediagnosisi
A.Tuberculosis B.ALL C.Hodgkin'slympho oma D.NonH Hodgkin'slym mphoma E.CLL

Answer:c)Hod dgkin'slymp phoma. The clinical fea e atures are suggestive in Hodgkin disease and histology demon ns e nstrates Ree edSternberg gcells,whicharepath hognomonic c.ReedSternbergcells sarecharacteristic bin nucleateor multinuclea atecellsfou undinHodg gkinsdisease(owlsey yenucleior rchurch plat tenuclei). Stag gingisviath heModified dAnnArbor rclassificati ion: IIn nvolvement tofasinglelymphnod deregionor rasingleextralymphaticsiteoror rgan. III Involvemen ntoftwoor rmorelymp phnodereg gionsonthe esameside eofthediaphragm (II)o oroneorm morelymphnoderegionsplusane extralympha aticsite(IIE E). IIIInvolvemen ntoflymphnodesonb bothsideso ofthediaph hragm. IV Involvementofoneormoreextr ralymphatic corgans(Lu ung,liver,b bonemarrow w,with orw withoutlymphnodeinv volvement).

Che x ray sh est how ing m mediastinal w idening due to lym mphadenop pathy in Ho odgkin's lymphoma

Dr.K KhalidYusuf fElzohrySo ohagTeachin ngHospital 2012 144

www.MRCPass.com

Haematology

HaematologyQ144: A 12 year old boy has recently been found to be anaemic and is undergoing investigations.Heisshortandhasanabnormalfacieswithfrontalandparietalbossing, enlargementofthemalar(maxillary)bonesandprotrudingteeth.Onexaminationofthe abdomenhehashepatosplenomegaly.Investigationsareasfollows: Hb7.5g/dl MCV65 Plt160x10^9/l Whichofthefollowingislikelytotreattheanaemia?
A.Ascorbicacid B.Ferroussulphate C.VitaminB12 D.Folicacid E.Bloodtransfusion

Answer:d)folicacid. The history of chronic anaemia and the examination features suggest a chronic haemolyticanaemiawithextramedullaryerythropoiesis.Thissuggeststhediagnosisof thalassaemia. In thalassaemia, folic acid supplementation is useful in treatment of anaemiaasthereisincreasedmetabolicdemandforfolicacid.

Dr.KhalidYusufElzohrySohagTeachingHospital2012 145

www.MRCPass.com

Haematology

HaematologyQ145: A30yearoldwomanisbleedingalotafterathyroidectomy.Investigationsshow: Hb11.3g/dl WBC5.2x10^9/l Plts230x10^9/l PT15sec(1316sec) APTT86sec(2838sec) APTT50:50mixwithnormalplasma37sec Whichofthefollowingisthemostlikelydiagnosis?


A.FactorVdeficiency B.Antiphospholipidsyndrome C.FactorVIIdeficiency D.FactorXIIdeficiency E.VonWillebrandsdisease

Answer:e)VonWillebrandsdisease. An isolatedprolonged APTT will becaused by deficiencies in factors VIII, IX, XI and XII andbyvonWillebrandsdisease. Antiphospholipid syndrome can cause a prolonged APTT but is not associated with bleedingandtheAPTTisnotcorrectedwithnormalplasma.FactorXandVdeficiency areassociatedwithbothaprolongedPTandAPTT. FactorVIIdeficiencyisassociatedwithaprolongedPT.

Dr.KhalidYusufElzohrySohagTeachingHospital2012 146

www w.MRCPass.c com

Haem matology

Hae ematologyQ Q146: A60 0yearoldm manpresen ntswithbac ckpains,ab bdominalpa ainsandpolyuria.Heh hasaHb of1 12g/dl,WCCof8x10^ ^9/l,pltof3 300x10^9/ /l.Serumca alciumis2.9 9(2.252.7) mmol/l andphosphate e1.2(0.88) )mmol/l. Wha atmightthebonemar rrowexamin nationshow w?
A.Increa asedblastce ells B.Increa asedproport tionofplasm macells C.Increa asedpromye elocytes D.Increasedinfiltrat tionoflymph haticcells E.Increa asedfibrosis

Answer:b)incr reasedprop portionofplasmacells. The ediagnosis ismultiple myeloma,a andthebonemarrow wshowsincr reasedamo ountsof plas smacells(>30%)assee enintheim magebelow.

147

Dr.K KhalidYusuf fElzohrySo ohagTeachin ngHospital 2012

www.MRCPass.com

Haematology

HaematologyQ147: A55yearoldmanhasacuteonsetconfusion,headache,nauseaandvomi ngandvisual disturbance. He had prescribed been prescribed NSAIDS and antibiotics for knee arthritis. On examination he was overweight, he looked plethoric, and cyanosed. There was 3 fingerbreadthshepatomegalyandthespleenwasenlargedabout5ngerbreadths. Investigationsshow: Hb21.2g/dl MCV71 WBC18x10^9/l Na135mmol/l K3.8mmol/l Urea6.2mmol/l Crea nine88micromoles/l Chloride105mmol/l Bicarbonate32mmol/l Calcium2.5 Albumin36g/l Phosphate0.9mmol/l ESR15mm/1sthour Whatisthebestmanagement?
A.Hyperbaricoxygen B.Splenectomy C.Prednisolone D.Broadspectrumantibiotics E.Venesection

Answer:e)venesection. The diagnosis is polycythaemia rubra vera. Increased serum viscosity may arise from hyperglobulinaemiaorfromanincreasedredcellmass,polycythaemia.Asaguideline, erythrocytosis should be suspected in men with a haemoglobin concentration greater than 18.0 g /L or in w omen with values greater than 17.0 g /L. Treatment of hyperviscositysyndromeshouldbewithfluidreplacementandvenesection. The cause of cyanosis is due to small vessel insufficiency and thrombosis, which will improvewithvenesection. Hydroxyurea and anagrelide are chemotherapeutic agents which can also be considered.

Dr.KhalidYusufElzohrySohagTeachingHospital2012 148

www.MRCPass.com

Haematology

HaematologyQ148: A35yearoldmanhasdiabetes.Onexaminationhealsohasaslategreydiscolouration aroundhisforearm.HehasaHbof13.5g/dl,plateletcount350x10^9/l,ASTof35U/l, ALPis120U/l,Albumin35g/l,ferri nis500g/l. Whichofthefollowingtestsismosthelpful?


A.Copperandcaeruloplasmin B.Transferrinsaturation C.Fastingglucose D.72hourfast E.Shortsynacthentest

Answer:b)transferrinsaturation. Thelikelydiagnosisishaemochromatosisduetothediabetes,pigmentation,andraised ferritin.Transferrinsaturationwouldberaisedinhaemochromatosis.

Dr.KhalidYusufElzohrySohagTeachingHospital2012 149

www w.MRCPass.c com

Haem matology

Hae ematologyQ Q149: A72 2yearoldw womanpre esentswith malaise,he eadachesandweaknessinherar rmsand legs Clinical e s. examination reveals ly n ymphadeno opathy and hepatosplenomegaly Nerve y. conductiontes stsshowasensoryneuropathy. HerbloodtestsrevealHb bof7.9g/dl, ,MCV95,WCC9x1 10^9/l.HerESRis80a andIgM para aproteinof18g/l(05) ). ichisthemostlikelydi iagnosis? Whi
A.Multi iplemyeloma B.CML C.CLL D.Waldenstrom'sm macroglobulin naemia E.AML

Answer:d)Waldenstrom's smacroglob bulinaemia. . InW Waldenstrom'smacrog globulinaem mia,increas sedserum proteinsleadstoava arietyof sym mptoms: Neu uropathy Hea adacheandfocalnervo oussystemi impairment t congestivecard diacfailure. . Diag gnosisisconfirmedbyhighIgMp paraproteinlevels(also oknownasaspike).

Apl lasmacello ontheblood dfilminWa aldenstrom' 's

Dr.K KhalidYusuf fElzohrySo ohagTeachin ngHospital 2012 150

www.MRCPass.com

Haematology

HaematologyQ150: A60yearoldwomanundergoesacolostomy,whichiscomplicatedbyapostoperative haemorrhage.Followingtransfusionof4unitsofblood,herhaemoglobinis12.g/dl.A weeklater,shehasthefollowingresults: Hb7.7g/dl WBC6.6x10^9/l Plts377x10^9/l MCV96 Bilirubin66umol/l DirectCoombstestpositive Whatisthemostlikelydiagnosis?
A.Furtherpostoperativebleed B.Subacuteendocarditis C.Delayedhaemolytictransfusionreaction D.Acutehaemolytictransfusionreaction E.Autoimmunehaemolyticanaemia

Answer:c)Delayedhaemolytictransfusionreaction. Themostlikelydiagnosisisadelayedhaemolytictransfusionreaction.Thesearedueto incompatibilities in red cell antigens other than the ABO groups. The antibodies are acquired rather than naturally occurring so they occur in patients who have been pregnant in the past or who have had blood transfusions. When stimulated by transfusion, an body levels increase over 710 days to cause a delayed haemolytic transfusionreaction.

Dr.KhalidYusufElzohrySohagTeachingHospital2012 151

www w.MRCPass.c com

Haem matology

Hae ematologyQ Q151: A65 5yearold ladypresen ntswithma alaiseandw weightloss, ,havingbeenreferred dbythe GPf forinvestigationofana aemia.Hisinvestigatio onsshow: MCV105 Hb8 8.0g/dl MCHC33g/dl( (3235) WCC11x1 10^9/l telets130x10^9/l urea6mo ol/l plat crea nine90 a mol/l sodium140 0mmol/l pota assium4mmol/l bilirubin18 8mol/l AST T28U/l ALP180U/ /l iron n50mol/l(1429) ferri n550 0g/l(1520 00) Bon marrow aspirate sh ne hows incre eased haem mosiderin, n normoblasti hyperpla ic asia and ring gedredbloo odcells Whi ichofthefo ollowingisl likely?
A.Acute emyeloidleu ukaemia B.Plasm macytoma C.Chron nicmyeloidleukaemia D.Sideroblasticanae emia E.Chron niclymphoblasticleukaem mia

Answer:d)side eroblasticanaemia. Ins sideroblasticanaemia, ,thereisin ncreasedbo onemarrow wiron.This isreflected dinthe incr reasediron storesinfe erritinand alsohaemo osiderinand dringedpre ematureredblood cells s(siderobla asts)dueto oexcessiro on.Siderobl lasticanaem miaoccurs duetoan enzyme defi iciency(AL LAsynthase e2deficienc cy),alcohol, ,drugs(ant tiTB),myelo odysplasia.

Ring gedsiderob blastsinaca aseofmyelo odysplasia

Dr.K KhalidYusuf fElzohrySo ohagTeachin ngHospital 2012 152

www.MRCPass.com

Haematology

HaematologyQ152: A 30 year old woman was sent to the hospital for investigation following a upper respiratorytractinfectionwhichwasslowtoresolve.Thefollowingresultswereseen: Hb11.5g/dl,MCV79,platelets650x10^9/l,WCC10x10^9/l,normalPTandAPTT. Whatisthelikelycauseofthethrombocytosis?
A.Polycythaemiarubravera B.Sideroblasticanaemia C.Reactivethrombocytosis D.Myelodysplasia E.Idiopathicthrombocytopenicpurpura

Answer:c)reactivethrombocytosis. Reactive thrombocytosis may be due to haemorrhage, iron deficiency, malignancy, infectionandconnectivetissuediseases.

Dr.KhalidYusufElzohrySohagTeachingHospital2012 153

www w.MRCPass.c com

Haem matology

Hae ematologyQ Q153: A75 5yearoldp patienthasb beenunwell.Herblood dtestsshow w: Hb7 7.0g/dl MCV V110 WCC4x10^9/ /l telets70x1 10^9/l plat urea a5mol/l crea nine110mol/l a sodium140mm mol/l assium4mmol/l pota The ebloodfilmshowsringsideroblast tswith15% %blastcells. . Bon nemarrows showhyper rcellularity Whi ichisthemostlikelydi iagnosis?
A.Acute emyeloidleu ukaemia B.Chron nicmyeloidleukaemia C.Myelo odysplastics syndrome D.Chron niclymphatic cleukaemia E.Nonh hodgkin'slym mphoma

Answer:c)mye elodysplasti icsyndrome e. Mye elodysplast tic syndro omes are associate ed with pancytopen nias along with g dyse erythropoie etic ringed sideroblas and bla cells in the peripheral circ sts ast n culation. Side eroblastswo ouldbeunlikelyinAM MLandCML, ,althoughb blastcellsco ouldbeseen.Ifthe bon marrow was hypo ne ocellular, th hen aplasti anaemia would be possible in this ic a e que estion.

Bon Marrow biopsy in myelodys ne w n splasia sho ing hy ow ypercellulari and ab ity bnormal meg gakaryocyte es
Dr.K KhalidYusuf fElzohrySo ohagTeachin ngHospital 2012 154

www.MRCPass.com

Haematology

HaematologyQ154: A45yearoldintravenousdruguseristreatedwithunfractionatedheparinforaDVTas heisbeingassessedforendocarditis.Twomonthspreviously,hehadreceivedheparin. A er3daysoftreatment,hisplateletcounthasfallenfromabaselineof180x10^9/lto 120x10^9/l.UponenquirytotheGP,hehadpreviouslyhadbloodtestswhichshowed anormalplateletcountandhehadnohistoryofbleedingproblems. Whichofthesestatementsistrue?


A.Deepveinthrombosis B.TypeIHIT C.Essentialthrombocytopenia D.Idiopathicthrombocytopenicpurpura E.HenochSchonleinPurpura

Answer:b)TypeIHIT. Themostlikelydiagnosisisheparininducedthrombocytopenia,inviewoftheprevious normalplateletcountsandrecentuseofheparin.Althoughthepatientbecomesmildly thrombocytopaenic, therapy can continue as it is likely to remain mild. In Type II HIT, antibodieswouldbemorereadilydetectablebyELISAandthethrombocytopeniawould bemoresevere.

Dr.KhalidYusufElzohrySohagTeachingHospital2012 155

www.MRCPass.com

Haematology

HaematologyQ155: A 30 year old woman attends A&E with marked breathlessness. Subsequently, a pulmonaryembolismisconfirmedbyV/Qscan.Bloodtestsrevealthefollowingresults: PT11/11s APTT67/31s(50:50mixtest:normalplasma55s) TT19/18s Hb10.2g/dl WCC1.8x10^9/l Plats90x10^9/l Whichoneofthefollowinginvestigationsismostappropriate?
A.Antiphospholipidantibody B.Bonemarrowexamination C.Clottingfactorlevels D.Testingforlupusanticoagulant E.Thrombophiliascreening

Answer:a)Antiphospholipidantibody. Thelupusanticoagulantisaformofantiphospholipidantibody,andthisislikelytobe presentasthereisprolongedAPTT.Thenamewasgiventotheantibodybecauseitwas firstfoundinpatientswithlupus.Thepresenseofthisislikelytohavepredisposedthe patient to a pulmonaryembolus. Although thrombophilia testing is indicated it isbest leftuntilaftertheinitialperiodofanticoagulation.

Dr.KhalidYusufElzohrySohagTeachingHospital2012 156

www.MRCPass.com

Haematology

HaematologyQ156: A 35 year old woman was admitted with breathlessness. On admission she looked cyanotic and was given high flow oxygen immediately. Arterial blood gas analysis revealedapHof7.40,PO2of11kPa,PCO2of4andoxygensatura onof50%byco oximeter. Whattreatmentshouldbegiven?
A.Nacetylcysteine B.Haemocdialysis C.Methyleneblue D.Oralactivatedcharcoal E.Ascorbicacid

Answer:c)Methyleneblue. Methaemoglobinaemia occurs when haemoglobin is oxidised and is unable to carry oxygen. Drugswhichcancausethisconditioninclude: antibiotics(dapsone,sulphonamidesandtrimethoprim) nitritesandnitrates localanaesthetics(lignocaineandprilocaine). PulseoximetersmeasurebothoxyHbandmetHb,thereforegivingfalsereassurancein patients with high levels of metHb. Methylene blue is used in those with severe poisoning or MetHb levels greater than 30%; excessive doses of methylene blue can themselves cause methaemoglobinaemia. Dapsone poisoning can be treated by activatedcharcoal,whichadsorbsit.

Dr.KhalidYusufElzohrySohagTeachingHospital2012 157

www.MRCPass.com

Haematology

HaematologyQ157: A 38 year old woman presents to the hospitalwith a history of headaches, decreased consciousnessandfevers.Herbloodresultsshow: Hb10.5g/dl WBC14x10^9/l Plts14x10^9/l Clottingscreennormal urea15mol/l crea nine210mol/l Whatisthemostlikelydiagnosis?
A.Acutelymphoblasticleukaemia B.Thromboticthrombocytopenicpurpura C.Disseminatedintravascularcoagulation D.Chronicmyeloidleukaemia E.Acutemyeloidleukaemia

Answer:b)Thromboticthrombocytopenicpurpura. Theclassic5featuresassociatedwithTTPare: fever thrombocytopenia microangiopathichaemolyticanaemia renalfailure neurologicalsymptoms

Dr.KhalidYusufElzohrySohagTeachingHospital2012 158

www w.MRCPass.c com

Haem matology

Hae ematologyQ Q158: A35 5yearoldm manpresen ntswitha1 10dayhisto oryoflethargy.Hehas samaculo papular rash handa1cm msizedpalp pablecervic callymphno ode.Investigationssho ow: Hb1 13.6g/dl WBC13.2x10^ ^9/l 9/l Plts280x10^9 odfilmshow wsreactivelymphocytes Bloo atisthemo ostlikelydia agnosis? Wha
A.Tuberculosis B.SLE C.Infect tiousmononucleosis D.Acute emyeloblast ticleukaemia a E.NonH Hodgkinsdis sease

Answer:c)Infe ectiousmon nonucleosis. . Com mmon causes of a rea active lymp phocytosis include infectious mononucleos (EBV sis infe ection),CMV Vinfectionandtoxopla asmainfection.

ReactiveLymphocytes

Dr.K KhalidYusuf fElzohrySo ohagTeachin ngHospital 2012 159

www.MRCPass.com

Haematology

HaematologyQ159: A 40 year old man presents with fatigue, weakness, and shortness of breath with exertionoverthepastfewdays. On examination, he is pale and jaundiced. His heart rate is 110 and blood pressure is 110/65mmHg.Hisbreathsoundswereclear,thereisasoftflowmurmuraudibleinthe aorticarea. Bloodresultsshow: Hb6.5g/dl MCV105 WCC6.2x10^9/l platelets250x10^9/l Reticulocytecount14%(0.5%to1.5%) AST27(131)U/l ALP78(20120)U/l Bilirubin65(122)mol/l lactatedehydrogenase410(105333)U/L Thebloodfilmshowsspherocytes. Whichtestshouldbeperformednext?
A.DirectCoombstest B.Osmoticfragility C.HAM'stest D.Bonemarrowaspirate E.Haemoglobinelectrophoresis

Answer:a)DirectCoombstest. Thediagnosisisautoimmunehaemolyticanaemia.Inhaemolyticanaemia,thebilirubin and LDH are raised, there is a reticulocytosis, and blood film shows spherocytes. Autoimmune haemolytic anaemia is associated with viral infections, drugs, lymphoproliferativediseasesandautoimmunediseases. ThedirectCoomb'stest(DAT)isusedtodetectIgGorC3boundtothesurfaceofthe redcell.Inpatientswithhemolysis,theDATisusefulindeterminingwhetherthereisan immune etiology. Immune causes of hemolysis, including autoimmune hemolytic anemias,druginducedhemolysis,anddelayedoracutehemolytictransfusionreactions, are characterized by a positive DAT. If the DAT test is negative, then hereditary spherocytosisshouldbeconsidered. Autoimmune haemolytic anaemia can be due to warm or cold reacting antibodies, dependingonthecauseoftheautoimmunereaction.WarmantibodiestendtobeIgG

Dr.KhalidYusufElzohrySohagTeachingHospital2012 160

www w.MRCPass.c com

Haem matology

andleadtosplenicdestru uctionofre edbloodcells,forming gspherocyte es.Coldant tibodies areusuallyIgM Mantibodies s.Theycaus seintravasc cularhaemo olysis.

Coo omb'stest

Dr.K KhalidYusuf fElzohrySo ohagTeachin ngHospital 2012 161

You might also like